You are on page 1of 57

Quadratic Equation

A man is like a fraction whose numerator is what he is and whose denominator is what he thinks of himself. The larger the denominator the
Tolstoy, Count Lev Nikolgevich
smaller the fraction.........
1. Polynomial :
A function f defined by f(x) = anxn + an – 1xn – 1 + .......+ a1x + a0
where a0, a1, a2, ......, an  R is called a polynomial of degree n with real coefficients (an  0, n  W).
If a0, a1, a2, ....., an  C, it is called a polynomial with complex coefficients.

2. Quadratic polynomial & Quadratic equation :


A polynomial of degree 2 is known as quadratic polynomial. Any equation f(x) = 0, where f is a quadratic
polynomial, is called a quadratic equation. The general form of a quadratic equation is
ax2 + bx + c = 0 .......(i)
Where a, b, c are real numbers, a  0.
If a = 0, then equation (i) becomes linear equation.
3. Difference between equation & identity :
If a statement is true for all the values of the variable, such statements are called as identities. If the
statement is true for some or no values of the variable, such statements are called as equations.
Example : (i) (x + 3)2 = x2 + 6x + 9 is an identity
(ii) (x + 3)2 = x2 + 6x + 8, is an equation having no root.
(iii) (x + 3)2 = x2 + 5x + 8, is an equation having – 1 as its root.
A quadratic equation has exactly two roots which may be real (equal or unequal) or imaginary.
a x2 + b x + c = 0 is:
 a quadratic equation if a  0 Two Roots
 a linear equation if a = 0, b  0 One Root
 a contradiction if a = b = 0, c  0 No Root
 an identity if a=b=c=0 Infinite Roots
If ax2 + bx + c = 0 is satisfied by three distinct values of ' x ', then it is an identity.
Example # 1 : (i) 3x2 + 2x – 1 = 0 is a quadratic equation here a = 3.
(ii) (x + 1)2 = x2 + 2x + 1 is an identity in x.
Solution : Here highest power of x in the given relation is 2 and this relation is satisfied by three different
values x = 0, x = 1 and x = – 1 and hence it is an identity because a polynomial equation of nth
degree cannot have more than n distinct roots.

4. Relation Between Roots & Co-efficients:


(i) The solutions of quadratic equation, a x2 + b x + c = 0, (a  0) is given by
 b  b2  4 a c
x=
2a
The expression, b2  4 a c  D is called discriminant of quadratic equation.
(ii) If ,  are the roots of quadratic equation,
a x2 + b x + c = 0 .......(i)
then equation (i) can be written as
a(x – ) (x – ) = 0
or ax2 – a( + )x + a  = 0 ......(ii)
equations (i) and (ii) are identical,
b coefficient of x
 by comparing the coefficients sum of the roots,  +  = – =–
a coefficient of x 2
c constant term
and product of the roots, = =
a coefficient of x 2
Corporate Office: CG Tower, A-46 & 52, IPIA, Near City Mall, Jhalawar Road, Kota (Raj.)-324005
Website : www.resonance.ac.in | E-mail : contact@resonance.ac.in
Toll Free : 1800 200 2244 | 1800 258 5555 | CIN: U80302RJ2007PLC024029 ADVQE - 1
Quadratic Equation
b c
(iii) Dividing the equation (i) by a, x2 + x+ =0
a a
b c
 x2 –  x + =0  x2 – ( + )x +  = 0
 a  a
 x2 – (sum of the roots) x + (product of the roots) = 0
Hence we conclude that the quadratic equation whose roots are  &  is x2 – ( + )x +  = 0

Example # 2 : If  and  are the roots of ax2 + bx + c = 0, find the equation whose roots are +2 and +2.
Solution : Replacing x by x – 2 in the given equation, the required equation is
a(x – 2)2 + b(x – 2) + c = 0 i.e., ax2 – (4a – b)x + (4a – 2b + c) = 0.

Example # 3 : The coefficient of x in the quadratic equation x 2 + px + q = 0 was taken as 17 in place of 13, its
roots were found to be – 2 and – 15. Find the roots of the original equation.
Solution : Here q = (– 2) × (– 15) = 30, correct value of p = 13. Hence original equation is
x2 + 13x + 30 = 0 as (x + 10) (x + 3) = 0
 roots are – 10, – 3

Self practice problems :


(1) If ,  are the roots of the quadratic equation cx2 – 2bx + 4a = 0 then find the quadratic equation
whose roots are
 
(i) , (ii) 2, 2 (iii)  + 1,  + 1
2 2
1  1    
(iv) , (v) ,
1  1    
(r  1)2 b2
(2) If r be the ratio of the roots of the equation ax2 + bx + c = 0, show that = .
r ac
Answers : (1) (i) cx2 – bx + a = 0 (ii) c2x2 + 4(b2 – 2ac)x + 16a2 = 0
2
(iii) cx – 2x(b + c) + (4a + 2b + c) = 0
(iv) (c – 2b + 4a)x2 + 2(4a – c) x + (c + 2b + 4a) = 0
(v) 4acx2 + 4(b2 – 2ac) x + 4ac = 0

5. Theory Of Equations :
If 1, 2, 3,......n are the roots of the equation;
f(x) = a 0x n + a 1x n-1 + a 2x n-2 +.... + an-1x + a n = 0 where a 0, a1,....,an are all real & a0  0 then,
a1 a2 a3 an
 1 =  a ,  1 2 = + a , 1 2 3 =  a ,....., 1 2 3........n = (1)n a
0 0 0 0

Note : (i) If  is a root of the equation f(x) = 0, then the polynomial f(x) is exactly divisible by (x ) or
(x ) is a factor of f(x) and conversely.
(ii) Every equation of nth degree (n 1) has exactly n roots & if the equation has more than n roots, it
is an identity.
(iii) If the coefficients of the equation f(x) = 0 are all real and  + i is its root, then   i is also a root.
i.e. imaginary roots occur in conjugate pairs.
(iv) An equation of odd degree will have odd number of real roots and an equation of even degree will
have even numbers of real roots.

(v) If the coefficients in the equation are all rational &  +  is one of its roots, then
  is also a root where ,   Q &  is not square of a rational number..
(vi) If there be any two real numbers 'a' & 'b' such that f(a) & f(b) are of opposite signs, then
f(x) = 0 must have odd number of real roots (also atleast one real root) between ' a ' and ' b '.
(vii) Every equation f(x) = 0 of degree odd has atleast one real root of a sign opposite to that of its
last term. (If coefficient of highest degree term is positive).

Corporate Office: CG Tower, A-46 & 52, IPIA, Near City Mall, Jhalawar Road, Kota (Raj.)-324005
Website : www.resonance.ac.in | E-mail : contact@resonance.ac.in
Toll Free : 1800 200 2244 | 1800 258 5555 | CIN: U80302RJ2007PLC024029 ADVQE - 2
Quadratic Equation
Example # 4 : If 2x3 + 3x2 + 5x + 6 = 0 has roots , ,  then find  +  + ,  + +  and .
Solution : Using relation between roots and coefficients, we get
3 5 6
+  +  = = – , +  +  = ,  = – = – 3.
2 2 2

Self practice problems :


(3) If 2p3 – 9pq + 27r = 0 then prove that the roots of the equations rx3 – qx2 + px – 1 = 0 are in H.P.

(4) If , ,  are the roots of the equation x3 + qx + r = 0 then find the equation whose roots are
(a)  + 2,  + , 
r r r
(b)  ,  , 
  
(c) 2, 2, 2
(d) 3, 3, 3
Answers : (4) (a) x3 + qx – r = 0 (b) x3 – qx2 – r2 = 0
(c) x3 + 2qx2 + q2 x – r2 = 0 (d) x3 – 3x2r + (3r2 + q3) x – r3 = 0

6. Nature of Roots:
Consider the quadratic equation, a x2 + b x + c = 0 having ,  as its roots;
D  b2  4 a c

D=0 D0
Roots are equal i.e.  =  =  b/2a Roots are unequal
& the quadratic expression can be expressed
as a perfect square of a linear polynomial

a, b, c  R & D > 0 a, b, c  R & D < 0


Roots are real Roots are imaginary  = p + i q,  = p  i q

a, b, c  Q & a, b, c  Q &
D is square of a rational number D is not square of a rational number
 Roots are rational  Roots are irrational
 i.e.  = p + q ,  = p  q
a = 1, b, c   & D is square of an integer
 Roots are integral.

Example # 5 : For what values of m the equation (1 + m) x2 – 2(1 + 3m)x + (1 + 8m) = 0 has equal roots.
Solution : Given equation is (1 + m) x2 – 2(1 + 3m)x + (1 + 8m) = 0 ........(i)
Let D be the discriminant of equation (i).
Roots of equation (i) will be equal if D = 0.
or 4(1 + 3m)2 – 4(1 + m) (1 + 8m) = 0
or 4(1 + 9m2 + 6m – 1 – 9m – 8m2) = 0
or m2 – 3m = 0 or, m(m – 3) = 0
 m = 0, 3.

Corporate Office: CG Tower, A-46 & 52, IPIA, Near City Mall, Jhalawar Road, Kota (Raj.)-324005
Website : www.resonance.ac.in | E-mail : contact@resonance.ac.in
Toll Free : 1800 200 2244 | 1800 258 5555 | CIN: U80302RJ2007PLC024029 ADVQE - 3
Quadratic Equation
Example # 6 : Find all the integral values of a for which the quadratic equation (x – a) (x – 10) + 1 = 0 has
integral roots.
Solution : Here the equation is x2 – (a + 10)x + 10a + 1 = 0. Since integral roots will always be rational it
means D should be a perfect square.
From (i) D = a2 – 20a + 96.
 D = (a – 10)2 – 4  4 = (a – 10)2 – D
If D is a perfect square it means we want difference of two perfect square as 4 which is possible
only when (a – 10)2 = 4 and D = 0.
 (a – 10) = ± 2  a = 12, 8

Example # 7 : If the roots of the equation (x – a) (x – b) – k = 0 be c and d, then prove that the roots of the
equation (x – c) (x – d) + k = 0, are a and b.
Solution : By given condition (x – a) (x – b) – k  (x – c) (x – d)
or (x – c) (x – d) + k  (x – a) (x – b)
Above shows that the roots of (x – c) (x – d) + k = 0 are a and b.

Example # 8 : Determine 'a' such that x2 – 11x + a and x2 – 14x + 2a may have a common factor.
Solution : Let x –  be a common factor of x2 – 11x + a and x2 – 14x + 2a.
Then x =  will satisfy the equations x2 – 11x + a = 0 and x2 – 14x + 2a = 0.
 2 – 11 + a = 0 and 2 – 14 + 2a = 0
Solving (i) and (ii) by cross multiplication method, we get a = 0, 24.

Example # 9 : Show that the expression x 2 + 2(a + b + c)x + 3(bc + ca + ab) will be a perfect square
if a = b = c.
Solution : Given quadratic expression will be a perfect square if the discriminant of its corresponding
equation is zero.
i.e. 4(a + b + c)2 – 4.3 (bc + ca + ab) = 0
or (a + b + c)2 – 3(bc + ca + ab) = 0
1
or ((a – b)2 + (b – c)2 + (c – a)2) = 0
2
which is possible only when a = b = c.

Self practice problems :


(5) For what values of 'k' the expression (4 – k)x2 + 2(k + 2)x + 8k + 1 will be a perfect square ?

(6) If (x – ) be a factor common to a1x2 + b1x + c and a2x2 + b2x + c, then prove that
(a1 – a2) = b2 – b1.

(7) If 3x2 + 2xy + 2y2 + 2ax – 4y + 1 can be resolved into two linear factors, Prove that '' is a root
of the equation x2 + 4ax + 2a2 + 6 = 0.

(8) Let 4x2 – 4( – 2)x +  – 2 = 0 (  R) be a quadratic equation. Find the values of '' for which
(i) Both roots are real and distinct.
(ii) Both roots are equal.
(iii) Both roots are imaginary
(iv) Both roots are opposite in sign.
(v) Both roots are equal in magnitude but opposite in sign.

(9) If P(x) = ax2 + bx + c, and Q(x) = – ax 2 + dx + c, ac  0 then prove that P(x) . Q(x) = 0 has
atleast two real roots.
Answers. (5) 0, 3
(8) (i) (– , 2)  (3, ) (ii)  {2, 3} (iii) (2, 3) (iv) (– , 2) (v) 

Corporate Office: CG Tower, A-46 & 52, IPIA, Near City Mall, Jhalawar Road, Kota (Raj.)-324005
Website : www.resonance.ac.in | E-mail : contact@resonance.ac.in
Toll Free : 1800 200 2244 | 1800 258 5555 | CIN: U80302RJ2007PLC024029 ADVQE - 4
Quadratic Equation
7. Graph of Quadratic Expression:
 the graph between x, y is always a parabola.

 b D 
 the coordinate of vertex are   2 a ,  4 a 
 
 If a > 0 then the shape of the parabola is concave upwards & if a < 0 then the shape of the parabola
is concave downwards.

 the parabola intersect the yaxis at point (0, c).


 the xcoordinate of point of intersection of parabola with xaxis are the real roots of the quadratic
equation f (x) = 0. Hence the parabola may or may not intersect the xaxis.

8. Range of Quadratic Expression f(x) = ax 2 + bx + c.


(i) Range :
 D 
If a > 0  f (x)   4 a ,  
 
 D
If a < 0  f (x)     ,  4 a 
 
D
Hence maximum and minimum values of the expression f (x) is  in respective cases and it
4a
b
occurs at x =  (at vertex).
2a
(ii) Range in restricted domain:
Given x  [x1, x2]
b
(a) If   [x1, x2] then,
2a
f (x)  [ min { f ( x1), f ( x 2 )} , max { f ( x1 ), f ( x 2 )}]

b
(b) If   [x1, x2] then,
2a
  D  D 
f (x)   min  f ( x1) , f (x 2 ) ,   , max  f ( x1) , f ( x 2 ) ,  
  4a  4 a  

9. Sign of Quadratic Expressions :


The value of expression f (x) = a x2 + b x + c at x = x0 is equal to ycoordinate of the point on parabola
y = a x2 + b x + c whose xcoordinate is x0. Hence if the point lies above the xaxis for some x = x0, then
f (x0) > 0 and viceversa.

Corporate Office: CG Tower, A-46 & 52, IPIA, Near City Mall, Jhalawar Road, Kota (Raj.)-324005
Website : www.resonance.ac.in | E-mail : contact@resonance.ac.in
Toll Free : 1800 200 2244 | 1800 258 5555 | CIN: U80302RJ2007PLC024029 ADVQE - 5
Quadratic Equation
We get six different positions of the graph with respect to xaxis as shown.

(i) Conclusions :
(a) a>0
(b) D>0
(c) Roots are real & distinct.
(d) f(x) > 0 in x  (– , )  (, )
(e) f(x) < 0 in x  (, )

(ii) (a) a>0


(b) D=0
(c) Roots are real & equal.
(d) f(x) > 0 in x  R – {}

(iii) (a) a>0


(b) D<0
(c) Roots are imaginary.
(d) f(x) > 0 x  R.

(iv) (a) a<0


(b) D>0
(c) Roots are real & distinct.
(d) f(x) < 0 in x  (– , )  (, )
(e) f(x) > 0 in x  (, )
(v) (a) a<0
(b) D=0
(c) Roots are real & equal.
(d) f(x) < 0 in x  R – {}

(vi) (a) a<0


(b) D<0
(c) Roots are imaginary.
(d) f(x) < 0 x  R.

Example # 10 : If c < 0 and ax2 + bx + c = 0 does not have any real roots then prove that
(i) a–b+c<0 (ii) 9a + 3b + c < 0.
Solution : c < 0 and D < 0  f(x) = ax2 + bx + c < 0 for all x  R
 f(– 1) = a – b + c < 0
and f(3) = 9a + 3b + c < 0

Example # 11 : Find the range of f(x) = x2 – 5x + 6.

D b  25  24  5 1
Solution : minimum of f(x) = – at x = – =–   at x = =–
4a 2a  4  2 4
maximum of f(x)  
 1 
Hence range is  ,   .
 4 

Corporate Office: CG Tower, A-46 & 52, IPIA, Near City Mall, Jhalawar Road, Kota (Raj.)-324005
Website : www.resonance.ac.in | E-mail : contact@resonance.ac.in
Toll Free : 1800 200 2244 | 1800 258 5555 | CIN: U80302RJ2007PLC024029 ADVQE - 6
Quadratic Equation

x2  x  4
Example # 12 : Find the range of rational expression y = if x is real.
x2  x  4

x2  x  4
Solution : y=  (y – 1)x 2 + (y + 1) x + 4(y – 1) = 0 ........(i)
x2  x  4

case- : if y  1, then equation (i) is quadratic in x


and  x is real
 D0  (y + 1)2 – 16(y – 1)2  0  (5y – 3) (3y – 5)  0 
3 5
y   ,  – {1}
5 3
case- : if y = 1, then equation becomes
2x = 0  x = 0 which is possible as x is real.
3 5
 Range  , 
5 3

x3
Example # 13 : Find the range of y = 2 , if x is real.
2x  3x  9

x3
Solution : y= 2
2x  3x  9
 2yx2 + (3y – 1)x + 3(3y – 1) = 0 .......(i)

case- : if y  0, then equation (i) is quadratic in x


 x is real
 D0
 (3y – 1)2 – 24y (3y – 1)  0
 (3y – 1) (21y + 1)  0
 1 1
y    ,  – {0}
 21 3 
case- : if y = 0, then equation becomes
x = –3 which is possible as x is real
 1 1
 Range y    , 
 21 3 
Self practice problems :
(10) If c > 0 and ax2 + 2bx + 3c = 0 does not have any real roots then prove that
(i) 4a – 4b + 3c > 0 (ii) a + 6b + 9c > 0 (iii) a + 2b + 6c > 0

(a  b)2
(11) If f(x) = (x – a) (x – b), then show that f(x)  – .
4
(12) Find the least integral value of 'k' for which the quadratic polynomial
(k – 1) x2 + 8x + k + 5 > 0  x  R.

x 2  34 x  71
(13) Find the range of the expression , if x is a real.
x 2  2x  7
mx 2  3 x  4
(14) Find the interval in which 'm' lies so that the expression can take all real
 4x 2  3x  m
values,x  R.
Answers : (12) k=4 (13) (– , 5]  [9, ) (14) m  [1, 7]
Corporate Office: CG Tower, A-46 & 52, IPIA, Near City Mall, Jhalawar Road, Kota (Raj.)-324005
Website : www.resonance.ac.in | E-mail : contact@resonance.ac.in
Toll Free : 1800 200 2244 | 1800 258 5555 | CIN: U80302RJ2007PLC024029 ADVQE - 7
Quadratic Equation
10. Location of Roots :
Let f (x) = ax² + bx + c, where a > 0 & a, b, c  R.

(i) (ii) (iii)

(i) Conditions for both the roots of f (x) = 0 to be greater than a specified number‘x 0’ are
b²  4ac  0 & f (x0) > 0 & ( b/2a) > x0.
(ii) Conditions for both the roots of f (x) = 0 to be smaller than a specified number ‘x 0’ are
b²  4ac  0 & f (x0) > 0 & ( b/2a) < x0.
(iii) Conditions for a number ‘x0’ to lie between the roots of f (x) = 0 is f (x0) < 0.

(iv) (v)

(iv) Conditions that both roots of f (x) = 0 to be confined between the numbers x 1 and
x2, (x1 < x2) are b²  4ac  0 & f (x1) > 0 & f (x2) > 0 & x1 < ( b/2a) < x2.
(v) Conditions for exactly one root of f (x) = 0 to lie in the interval (x1, x2) i.e.
x1 < x < x2 is f (x1). f (x2) < 0.
Example # 14 : Let x2 – (m – 3) x + m = 0 (m  R) be a quadratic equation, then find the values of 'm' for which
(a) both the roots are greater than 2.
(b) both roots are positive.
(c) one root is positive and other is negative.
(d) One root is greater than 2 and other smaller than 1
(e) Roots are equal in magnitude and opposite in sign.
(f) both roots lie in the interval (1, 2)

Solution : (a)

Condition - : D0  (m – 3)2 – 4m  0  m2 – 10m + 9  0


 (m – 1) (m – 9)  0
 m  (– , 1]  [9, ) ......(i)

Condition - : f(2) > 0  4 – (m – 3)2 + m > 0 m < 10 ......(ii)


b m3
Condition - : – >2  2  m>7 .....(iii)
2a 2
Intersection of (i), (ii) and (iii) gives m  [9, 10)

(b)

Condition -  D0  m  (– , 1] [9, )


Condition -  f(0) > 0  m>0
b m3
Condition -   >0  >0  m>3
2a 2
intersection gives m  [9, ) Ans.
Corporate Office: CG Tower, A-46 & 52, IPIA, Near City Mall, Jhalawar Road, Kota (Raj.)-324005
Website : www.resonance.ac.in | E-mail : contact@resonance.ac.in
Toll Free : 1800 200 2244 | 1800 258 5555 | CIN: U80302RJ2007PLC024029 ADVQE - 8
Quadratic Equation

(c)

Condition -  f(0) < 0  m<0 Ans.

(d)

Condition -  f(1) < 0  4<0  m 


Condition -  f(2) < 0  m > 10
Intersection gives m Ans.
(e) sum of roots = 0  m=3
and f(0) < 0  m<0
 m  Ans.

(f)

Condition -  D  0  m  (– , 1]  [9, )


Condition -  f(1) > 0  1 – (m – 3) + m > 0  4 > 0which is true  m  R
Condition -  f(2) > 0  m < 10
b m3
Condition - V 1 < – <2  1< <2  5<m<7
2a 2
intersection gives m  Ans.

Example # 15 : Find all the values of 'a' for which both the roots of the equation (a – 2)x 2 – 2ax + a = 0 lies
in the interval (– 2, 1).
Solution : Case-I : f(–2) > 0  4(a – 2) + 4a + a > 0
8
9a – 8 > 0  a>
9
f(1) > 0  a – 2 – 2a + a > 0
– 2 > 0 not possible
 a 
Case-II : a–2<0  a<2
8
f(–2) < 0  a<
9
f(1) < 0  aR
b 4
–2<– <1  a<
2a 3
D0  a0
 8
intersection gives a  0, 
 9
 8
complete solution a  0,   {2}
 9

Corporate Office: CG Tower, A-46 & 52, IPIA, Near City Mall, Jhalawar Road, Kota (Raj.)-324005
Website : www.resonance.ac.in | E-mail : contact@resonance.ac.in
Toll Free : 1800 200 2244 | 1800 258 5555 | CIN: U80302RJ2007PLC024029 ADVQE - 9
Quadratic Equation
Self practice problems :

(15) Let x2 – 2(a – 1)x + a – 1 = 0 (a  R) be a quadratic equation, then find the value of 'a' for which
(a) Both the roots are positive (b) Both the roots are negative
(c) Both the roots are opposite in sign. (d) Both the roots are greater than 1.
(e) Both the roots are smaller than 1.
(f) One root is small than 1 and the other root is greater than 1.

(16) Find the values of p for which both the roots of the equation 4x 2 – 20px + (25p2 + 15p – 66) = 0
are less than 2.

(17) Find the values of '' for which 6 lies between the roots of the equation x2 + 2( – 3)x + 9 = 0.

(18) Let x2 – 2(a – 1)x + a – 1 = 0 (a  R) be a quadratic equation, then find the values of 'a' for
which
(i) Exactly one root lies in (0, 1). (ii) Both roots lies in (0, 1).
(iii) At least one root lies in (0, 1).
(iv) One root is greater than 1 and other root is smaller than 0.

(19) Find the values of a, for which the quadratic expression ax 2 + (a – 2) x – 2 is negative for
exactly two integral values of x.

Answers : (15) (a) [2, ) (b)  (c) (– , 1) (d)  (e) (– , 1] (f) (2, )

 3
(16) (– , –1) (17)   ,  
 4
(18) (i) (– , 1)  (2, ) (ii)  (iii) ( – , 1)  (2, ) (iv) 
(19) [1, 2)

11. Common Roots:


Consider two quadratic equations, a1 x2 + b1 x + c1 = 0 & a2 x2 + b2 x + c2 = 0.
(i) If two quadratic equations have both roots common, then the equations are identical and their
co-efficient are in proportion.
a1 b c
i.e. = 1 = 1 .
a 2 b2 c 2
(ii) If only one root is common, then the common root '  ' will be :
c a  c 2 a1 b c  b2 c1
= 1 2 = 1 2
a 1 b2  a 2 b1 c1 a 2  c2 a1
Hence the condition for one common root is :

 c1 a 2  c2 a1 2 = a1 b 2  a 2 b1  b1 c2  b2 c1 


Note : If f(x) = 0 & g(x) = 0 are two polynomial equation having some common root(s) then those common root(s)
is/are also the root(s) of h(x)  a f(x) + bg (x) = 0.

Example # 16 : If x2 – ax + b = 0 and x2 – px + q = 0 have a root in common and the second equation has
ap
equal roots, show that b + q = .
2
2
Solution : Given equations are : x – ax + b= 0 ........ (i)
and x2 – px + q = 0. ........ (ii)
Let  be the common root. Then roots of equation (ii) will be  and . Let  be the other root of
equation (i). Thus roots of equation (i) are ,  and those of equation (ii) are , .
Now +=a ........ (iii)

Corporate Office: CG Tower, A-46 & 52, IPIA, Near City Mall, Jhalawar Road, Kota (Raj.)-324005
Website : www.resonance.ac.in | E-mail : contact@resonance.ac.in
Toll Free : 1800 200 2244 | 1800 258 5555 | CIN: U80302RJ2007PLC024029 ADVQE - 10
Quadratic Equation
 = b ........ (iv)
2 = p ........ (v)
2 = q ........ (vi)
L.H.S. = b + q =  + 2 = ( + ) ........ (vii)
ap (  ) 2
and R.H.S. = = =  ( + ) ........ (viii)
2 2
from (vii) and (viii), L.H.S. = R.H.S.

Example # 17 : If a, b, c  R and equations ax2 + bx + c = 0 and x2 + 2x + 9 = 0 have a common root, show that
a : b : c = 1 : 2 : 9.
Solution : Given equations are : x2 + 2x + 9 = 0 ........(i)
and ax2 + bx + c = 0 ........(ii)
Clearly roots of equation (i) are imaginary since equation (i) and (ii) have a common root,
therefore common root must be imaginary and hence both roots will be common.
Therefore equations (i) and (ii) are identical
a b c
 = =  a:b:c=1:2:9
1 2 9

Self practice problems :


(20) If the equations ax2 + bx + c = 0 and x3 + x – 2 = 0 have two common roots then show that
2a = 2b = c.
a b c
(21) If ax2 + 2bx + c = 0 and a1x2 + 2b1x + c1 = 0 have a common root and , , are in A.P..
a1 b1 c1
show that a1, b1, c1 are in G.P.

12. Graphs of Polynomials

y = anxn + ............ + a1x + a0. The points where y1 = 0 are called turning points which are critial in plotting
the graph.

Example # 18 : Draw the graph of y = 2x3 – 15x2 + 36x + 1


Solution. y = 6x2 – 30x + 36 = 6(x – 3) (x – 2)

x 2 3  –

y 29 28  –

Example # 19 : Draw the graph of y = 3x4 + 4x3 + 3,


Solution. y = –12x3 + 12x
y = –12x2 (x – 1)

x 0 1  –

y 3 4 – –

Corporate Office: CG Tower, A-46 & 52, IPIA, Near City Mall, Jhalawar Road, Kota (Raj.)-324005
Website : www.resonance.ac.in | E-mail : contact@resonance.ac.in
Toll Free : 1800 200 2244 | 1800 258 5555 | CIN: U80302RJ2007PLC024029 ADVQE - 11
f}?kkr lehdj.k

A man is like a fraction whose numerator is what he is and whose denominator is what he thinks of himself. The larger the denominator the
smaller the fraction......... Tolstoy, Count Lev Nikolgevich

1. cgq i n (Polynomial) :
dksbZ Qyu f bl çdkj ifjHkkf"kr gks fd f(x) = an x n + an – 1x n – 1 + .......+ a1x + a0
tgk¡ a0, a1, a2,...,an R rks ;g Qyu f, okLrfod xq. kkadks okyk n ?kkr dk cgqin dgykrk gS tcfd an  0, n  W
;fn a0, a1, a2, ....., an  C gks] rks ;g lfEeJ xq. kkadksa okyk cgqin dgykrk gSA
2. f}?kkr cgq i n ,oa f}?kkr lehdj.k (Quadratic polynomial & Quadratic equation):
nks ?kkr dk ,d cgqin f}?kkr cgqin dgykrk gSA dksbZ lehdj.k f(x) = 0, tgk¡ f f}?kkr cgqin gS] f}?kkr lehdj.k
dgykrh gSA f}?kkr lehdj.k dk O;kid :i fuEu gS&
ax 2 + bx + c = 0 .......(i)
tgk¡ a, b, c okLrfod la[ ;k,¡ gS tcfd a  0. ;fn a = 0 gks] rks lehdj.k (i) jSf[kd lehdj.k cu tkrk gSA
3. lehdj.k ,oa loZ l fedk es a vUrj % (Difference between equation & identity) :
;fn dksbZ dFku pj ds lHkh ekuksa ds fy, lR; gks] rks dFku loZl fedk dgykrk gSA ;fn dFku pj ds dqN gh ekuksa
ds fy, lR; gks] rks dFku lehdj.k dgykrk gSA
mnkgj.k : (i) (x + 3)2 = x 2 + 6x + 9 ,d loZ l fedk gS A
(ii) (x + 3)2 = x 2 + 6x + 8 ,d lehdj.k gS ftldk dks bZ ew y ugha gS A
(iii) (x + 3)2 = x 2 + 5x + 8 ,d lehdj.k gS ftldk ,d ew y – 1 gS A
,d f}?kkr lehdj.k ds Bhd nks ewy gksrs gS tks okLrfod (leku ;k vleku) ;k dkYifud gks ldrs gSA
a x 2 + b x + c = 0 ——
 ,d f}?kkr lehdj.k gksxh ;fn a0 nks ewy
 jSf[kd lehdj.k gksxh ;fn a = 0, b  0 ,d ewy
 fojks/kkHkkl gS ;fn a = b = 0, c  0 dksbZ ewy ugha
 loZl fedk gksxh ;fn a=b=c=0 vuUr ewy
;fn a x 2 + b x + c = 0, x ds rhu fHkUu&fHkUu ekuksa ls lUrq"V gks] rks ;g ,d loZl fedk gksxhA
mnkjg.k # 1 : (i) 3x 2 + 2x – 1 = 0 ,d f}?kkr lehdj.k gS ] tgk¡ a = 3 gS A
(ii) (x + 1)2 = x 2 + 2x + 1, x es a ,d loZ l fedk gS A
gy : ;gk¡ fn;s x;s lac U/k esa x dh vf/kdre ?kkr 2 gS ,oa ;g lac U/k x ds rhu fHkUu&fHkUu ekuksa x= 0, x = 1
vkSj x = – 1 ls laUrq"V gksrk gSA vr% ;g ,d loZl fedk gS D;ksafd ,d n ?kkr ds cgqin lehdj.k ds n
ls T;knk fHkUu&fHkUu ewy ugha gks ldrs gSA
4. ew yks a rFkk xq . kka d ks es a lEcU/k (Relation Between Roots & Co-efficients) :
 b  b2  4 a c
(i) f}?kkr lehdj.k a x 2 + b x + c = 0, (a  0) ds gy x = }kjk fn;s tkrs gSA
2a
O;atd b2  4 a c  D, f}?kkr lehdj.k dk foospd dgykrk gSA
(ii) ;fn f}?kkr lehdj.k a x 2 + b x + c = 0 .......(i)
ds ewy ,  gks] rks lehdj.k (i) dks a(x – ) (x – ) = 0 }kjk fy[kk tk ldrk gSA
;k ax 2 – a( + )x + a  = 0 ......(ii)
lehdj.k (i) ,oa (ii) ,d leku gSA
 xq. kkadksa dh rqyuk djus ij
b x dk xq.kkad c vpj in
ewyksa dk ;ksx =  +  = – =– 2
x dk xq.kkad
rFkk ewyksa dk xq. kuQy = = 2
x dk xq.kkad
a a

Corporate Office: CG Tower, A-46 & 52, IPIA, Near City Mall, Jhalawar Road, Kota (Raj.)-324005
Website : www.resonance.ac.in | E-mail : contact@resonance.ac.in
Toll Free : 1800 200 2244 | 1800 258 5555 | CIN: U80302RJ2007PTC024029 ADVQE - 1
f}?kkr lehdj.k
(iii) lehdj.k (i) dks a ls foHkkftr djus ij
b c b c
x2 + x+ =0 x2 – 
 x + =0
a a  a  a
 x 2 – ( + )x +  = 0  x 2 – (ew yks a dk ;ks x) x + (ew yks a dk xq . kuQy) = 0
blls fu"d"kZ fudyrk gS fd f}?kkr lehdj.k ftlds ewy  ,oa  gks] x 2 – ( + )x +  = 0 gksrh gSA
mnkjg.k # 2 : ;fn ax 2 + bx + c = 0 ds nks ewy  vkSj  gks] rks og lehdj.k Kkr dhft, ftlds ewy + 2 vkSj +
2 gS A
gy fn;s x;s lehdj.k esa x dks (x – 2) ls izfrLFkkfir djus ij] vHkh"V lehdj.k gS&
a(x – 2)2 + b(x – 2) + c = 0 vFkkZr~ ax 2 – (4a – b)x + (4a – 2b + c) = 0.
mnkjg.k # 3 : lehdj.k x 2 + px + q = 0 esa x ds xq.kkad dks xyrh ls 13 dh txg 17 fy[kus ij ewy – 2 vkSj – 15 izkIr
gksrs gS rks ewy ¼lgh½ lehdj.k ds ewy Kkr dhft,A
gy ;gk¡ q = (– 2) × (– 15) = 30, p dk lgh eku = 13. vr% ewy ¼lgh½ lehdj.k gS&
x 2 + 13x + 30 = 0  (x + 10) (x + 3) = 0
 ewy – 10, – 3
vE;kl dk;Z :
(1) ;fn ,  f}?kkr lehdj.k cx2 – 2bx + 4a = 0 ds ewy gks] rks og f}?kkr lehdj.k Kkr dhft, ftlds ewy
gS &
 
(i) , (ii) 2 ,  2 (iii)  + 1,  + 1
2 2
1  1    
(iv) , (v) ,
1  1    
(r  1)2 b2
(2) ;fn lehdj.k ax 2 + bx + c = 0 ds ewyksa dk vuqikr r gks rks fl} dhft, fd = .
r ac
mÙkj : (1) (i) cx2 – bx + a = 0 (ii) c2x2 + 4(b2 – 2ac)x + 16a2 = 0
2
(iii) cx – 2x(b + c) + (4a + 2b + c) = 0
(iv) (c – 2b + 4a)x2 + 2(4a – c) x + (c + 2b + 4a) = 0
(v) 4acx2 + 4(b2 – 2ac) x + 4ac = 0

5. lehdj.k fl}kUr (Theory of Equations) :


;fn f(x) = a0x n + a1x n-1 + a2x n-2 +.... + an-1x + an = 0, tgk¡ a0, a1,.... an lHkh okLrfod gS rFkk a0  0, ds ewy
a1 a2 a3 an
1, 2, 3, ......n gks ] rks  1 =  ,  1 2 = + , 1 2 3 =  ,....., 1 2 3. .......n = (1)n
a0 a0 a0 a0
uks V : (i) ;fn lehdj.k f(x) = 0 dk ,d ewy ''gks] rks cgqin f(x), (x ) ls iw. kZr% foHkkftr gksrk gS ;k f(x) dk
,d xq. ku[k.M (x ) gksxkA blh çdkj bldk foijhr Hkh lR; gSA
(ii) n (n 1) ?kkr dh iz R;s d lehdj.k ds Bhd n ew y gksr s gS rFkk ;fn lehdj.k ds ewy n ls vf/kd gS] rc ;g
,d loZl fedk dgykrh gSA
(iii) ;fn f(x) = 0 ds lHkh xq. kkad okLrfod gks ,oa bldk ,d ewy  + i gks] rks   i Hkh bldk ,d ewy gksxk
vFkkZr dkYifud ewy la; qXeh ;qXeksa esa gksrs gSA
(iv) ,d fo"ke ?kkr dh lehdj.k ds okLrfod ewyksa dh la[;k fo"ke gksxh rFkk le ?kkr dh lehdj.k ds okLrfod
ewyksa dh la[ ;k le gksxhA
(v) ;fn lehdj.k ds lHkh xq. kkad ifjes; gS rFkk bldk ,d ewy  +  gks] rks   Hkh bldk ewy gksxk
tgk¡ ,   Q rFkk  ifjes; la[ ;k dk oxZ ugha gSA
(vi) ;fn dksbZ nks okLrfod la[ ;k,¡ a ,oa b bl izdkj gks fd f(a) ,oa f(b) foijhr fpUg ds gks] rks a ,oa b ds
e/;] f(x) = 0 ds okLrfod ewyksa dh la[ ;k fo"ke ¼de ls de ,d okLrfod ewy½ gksxhA
(vii) fo"ke ?kkr dh izR;sd lehdj.k f(x) = 0 dk ,d okLrfod ewy bl lehdj.k ds vfUre in ds fpUg ds foijhr
fpUg dk gksrk gSA ¼;fn vf/kdre ?kkr ds in dk xq. kkad /kukRed gksA).
Corporate Office: CG Tower, A-46 & 52, IPIA, Near City Mall, Jhalawar Road, Kota (Raj.)-324005
Website : www.resonance.ac.in | E-mail : contact@resonance.ac.in
Toll Free : 1800 200 2244 | 1800 258 5555 | CIN: U80302RJ2007PTC024029 ADVQE - 2
f}?kkr lehdj.k
mnkgj.k # 4 : ;fn 2x 3 + 3x 2 + 5x + 6 = 0 ds ewy , ,  gks] rks rc  +  + ,  + +  ,oa Kkr dhft,A
gy : ewyksa rFkk xq. kkadksa esa lEcU/k ds ç;ksx ls
3 5 6
+  +  = = – , +  +  = ,  = – = – 3.
2 2 2

vE;kl dk;Z :
(3) ;fn 2p3 – 9pq + 27r = 0 gks] rks fl) dhft, fd lehdj.k rx3 – qx2 + px – 1 = 0 ds ewy gjkRed Js.kh esa gSA
(4) ;fn lehdj.k x3 + qx + r = 0 ds ewy , ,  gks] rks rc og lehdj.k Kkr dhft, ftlds ewy fuEu gks&
(a)  + 2,  + , 
r r r
(b)  ,  , 
  
(c) 2, 2, 2

(d) 3, 3, 3

Answers : (4) (a) x3 + qx – r = 0 (b) x3 – qx2 – r2 = 0


(c) x3 + 2qx2 + q2 x – r2 = 0 (d) x3 – 3x2r + (3r2 + q3) x – r3 = 0

6. ew yks a dh iz d ` fr (Nature of Roots) :


ekuk a x 2 + b x + c = 0 ,d f}?kkr lehdj.k gS ftlds ewy ,  gSA
D  b2  4 a c

D=0 D0
ewy leku gS] vFkkZr~ =  =  b/2a ewy vleku gSA
rFkk f}?kkr O;atd dks ,d jSf[kd cgqin ds
iw. kZ oxZ ds :i esa O;Dr fd;k tk ldrk gSA

a, b, c  R, ,oa D > 0 a, b, c  R, ,oa D < 0


ewy okLrfod gSaA ewy dkYifud gSa] vFkkZr~  = p + i q,  = p  i q

a, b, c  Q ,oa a, b, c  Q ,oa
D ,d ifjes ; la [ ;k dk iw . kZ oxZ gS A D ,d ifjes ; la [ ;k dk iw . kZ oxZ ugha gS A
ew y ifjes ; gS a A  ew y vifjes ; gS A
 vFkkZr~  = p + q ,  = p  q
a = 1 vkS j b, c   ,oa D ,d iw . kk± d la [ ;k dk iw . kZ oxZ gS A  ewy iw. kkaZd gSA

mnkjg.k # 5 : 'm' ds fdu ekuksa ds fy;s lehdj.k (1 + m) x 2 – 2(1 + 3m)x + (1 + 8m) = 0 ds ewy cjkcj gSA
gy : nh xbZ lehdj.k (1 + m) x 2 – 2(1 + 3m)x + (1 + 8m) = 0 ........(i)
ekuk lehdj.k (i) dk foospd D gSA lehdj.k (i) ds ewy cjkcj gksxsa ;fn D = 0 gksA
vFkkZr~ 4(1 + 3m)2 – 4(1 + m) (1 + 8m) = 0
vFkkZr~ 4(1 + 9m 2 + 6m – 1 – 9m – 8m 2) = 0
vFkkZr~ m 2 – 3m = 0 ;k m(m – 3) = 0
 m = 0, 3.

Corporate Office: CG Tower, A-46 & 52, IPIA, Near City Mall, Jhalawar Road, Kota (Raj.)-324005
Website : www.resonance.ac.in | E-mail : contact@resonance.ac.in
Toll Free : 1800 200 2244 | 1800 258 5555 | CIN: U80302RJ2007PTC024029 ADVQE - 3
f}?kkr lehdj.k
mnkjg.k # 6 : 'a' ds osa lHkh iw.kk±d eku Kkr dhft, ftlds fy;s lehdj.k (x – a) (x – 10) + 1 = 0 ds ewy iw.kk±d gksA
gy : nh xbZ lehdj.k x 2 – (a + 10)x + 10a + 1 = 0 gSA pwafd iw. kk±d ewy lnSo ifjes; gksrs gS vFkkZr D iw. kZ
oxZ gksuk pkfg,A (i) ls D = a2 – 20a + 96.
 D = (a – 10)2 – 4  4 = (a – 10)2 – D
ysfdu D ,d iw. kZ oxZ gS vFkkZr ge pkgrs gS fd nks iw. kZ oxksZ dk vUrj 4 gks tks dso y rHkh laHko gS tc
(a – 10)2 = 4 ,oa D = 0 gks A
 (a – 10) = ± 2  a = 12, 8

mnkjg.k # 7 : ;fn lehdj.k (x – a) (x – b) – k = 0 ds ewy 'c' vkS j 'd' gks rks fl} dhft, fd lehdj.k
(x – c) (x – d) + k = 0 ds ew y 'a' vkS j 'b' gS A
gy : fn, x;s izfrcU/k ls (x – a) (x – b) – k  (x – c) (x – d) ;k (x – c) (x – d) + k  (x – a) (x – b)
vr% blls fl) gksrk gS fd lehdj.k (x – c) (x – d) + k = 0 ds ewy a vkSj b gSA

mnkgj.k # 8 : 'a' dk og eku Kkr dhft, ftlds fy, x 2 – 11x + a ,oa x 2 – 14x + 2a dk ,d xq. ku[k.M mHk;fu"B gSA
gy : ekuk x 2 – 11x + a ,oa x 2 – 14x + 2a dk ,d mHk;fu"B xq. ku[k.M x –  gSA
rc x =  lehdj.k x 2 – 11x + a = 0 ,oa x 2 – 14x + 2a = 0 dks lUrq"V djsxkA
 2 – 11 + a = 0 ,oa 2 – 14 + 2a = 0
lehdj.k (i) ,oa (ii) dks otz xq. ku fof/k ls gy djus ij a = 0, 24.

mnkgj.k # 9 : iznf'kZr dhft, fd O;atd x 2 + 2(a + b + c)x + 3(bc + ca + ab) ,d iw. kZ oxZ gS ;fn a = b = c.
gy : fn;k x;k O;atd iw. kZ oxZ gksxk ;fn blds laxr lehdj.k dk foospd 'kwU; gksA
vFkkZr~ 4(a + b + c)2 – 4.3 (bc + ca + ab) = 0 ;k (a + b + c)2 – 3(bc + ca + ab) = 0
1
;k ((a – b)2 + (b – c)2 + (c – a)2) = 0 tks dso y rHkh laHko gS tc a = b = c gksA
2

vH;kl dk;Z :
(5) 'k' ds fdl eku ds fy, O;a td (4 – k)x 2 + 2(k + 2)x + 8k + 1 ,d iw . kZ oxZ gks xk \

(6) ;fn O;atd a1x 2 + b1x + c rFkk a2x 2 + b2x + c dk mHk;fu"B xq. ku[k.M (x –  gks] rks fl) dhft, fd
(a1 – a2) = b2 – b1.

(7) ;fn 3x 2 + 2xy + 2y2 + 2ax – 4y + 1 dks nks js[ kh; xq. ku[k.Mks ds :i esa fy[kk tk ldrk gks] rks fl)
dhft, fd lehdj.k x 2 + 4ax + 2a2 + 6 = 0 dk ,d ewy '' gSA

(8) ekukfd 4x 2 – 4( – 2)x +  – 2 = 0 (  R), ,d f}?kkr lehdj.k gSA '' ds os eku Kkr dhft, ftuds
fy;s
(i) nksuks ewy okLrfod vkSj fHkUu gksA (ii) nksuksa ewy leku gksA
(iii) nksuksa ewy dkYifud gksA (iv) nksuksa ewyks ds fpUg foijhr gksA
(v) nksuksa ewyksa dk ifjek.k cjkcj gks ,oa fpUg foijhr gksA

(9) ;fn P(x) = ax 2 + bx + c, rFkk Q(x) = – ax 2 + dx + c, ac  0 gks ] rks fl} dhft, fd


P(x) . Q(x) = 0 ds de ls de nks ew y okLrfod gS a A
Answers. (5) 0, 3
(8) (i) (– , 2)  (3, ) (ii)  {2, 3} (iii) (2, 3) (iv) (– , 2) (v) 

Corporate Office: CG Tower, A-46 & 52, IPIA, Near City Mall, Jhalawar Road, Kota (Raj.)-324005
Website : www.resonance.ac.in | E-mail : contact@resonance.ac.in
Toll Free : 1800 200 2244 | 1800 258 5555 | CIN: U80302RJ2007PTC024029 ADVQE - 4
f}?kkr lehdj.k
7. f}?kkr O;a t d dk vkys [ k (Graph of Quadratic Expression) :
2
 D   b 
2
y = f (x) = a x + b x + c ;k y   = a  x  
 4a   2 a 
 x, y es a vkys [ k lnS o ,d ijoy; gS A
 b D 
 'kh"kZ ds funsZ'kkad   2 a ,  4 a  gSA
 
 ;fn a > 0 gks] rks ijoy; dk vkdkj mij dh vksj vory gS rFkk ;fn a < 0 gks] rks ijoy; dk vkdkj uhps
dh vksj vory gSA

 ijoy; yv{k dks fcUnq (0, c) ij izfrPNsn djrk gSA


 ijoy; ,oa xv{k ds izfrPNsn fcUnqvksa ds x-funsZ'kkad f}?kkr lehdj.k f (x) = 0 ds okLrfod ewy gksrs gSA
vr% ijoy; x v{k dks izfrPNsn dj ldrk gS vkSj ugha HkhA
8. f}?kkr O;a t d f(x) = a x 2 + b x + c dk ifjlj
(Range of Quadratic Expression f(x) = ax 2 + bx + c)
(i) ifjlj %
 D 
;fn a>0  f (x)   4 a ,  
 

 D
;fn a<0  f (x)     ,  4 a 
 

D b
O;atd f (x) dk vf/kdre ,oa U;wure eku laxr fLFkfr;ksa esa  gksrk gS ,oa ;g x =  ('kh"kZ ij )
4a 2a
ij izkIr gksrk gSA
(ii) iz f rcfU/kr iz k Ur es a ifjlj %
fn;k x;k gS fd x  [x 1, x 2],
b
(a) ;fn   [x 1, x 2] gks ] rks f (x)  [min {f(x 1), f(x 2)}, max{f(x 1), f(x 2)}]
2a

b   D   D 
(b) ;fn   [x 1, x 2] gks ] rks f (x)  min  f ( x1) , f ( x 2 ) ,   , max  f ( x1 ) , f ( x 2 ) ,  
2a   4 a   4 a 

9. f}?kkr O;a td dk fpUg (Sign of Quadratic Expressions) :


x = x 0 ij O;a td f (x) = a x 2 + b x + c dk eku ijoy; y = a x 2 + b x + c ij fLFkr fcUnq ftldk x–funs Z ' kka d
x 0 gS , ds y–funs Z ' kka d ds cjkcj gks r k gS A vr% ;fn fdlh x = x 0 ds fy, fcUnq x–v{k ls mij fLFkr gks ] rks
f (x 0) > 0 ,oa x v{k ls uhps fLFkr gks ] rks f(x 0) < 0

Corporate Office: CG Tower, A-46 & 52, IPIA, Near City Mall, Jhalawar Road, Kota (Raj.)-324005
Website : www.resonance.ac.in | E-mail : contact@resonance.ac.in
Toll Free : 1800 200 2244 | 1800 258 5555 | CIN: U80302RJ2007PTC024029 ADVQE - 5
f}?kkr lehdj.k
x–v{k ds lkis { k ijoy; dh fuEukuq l kj N% fLFkfr;k¡ lEHko gS &

(i) fu"d"kZ
(a) a>0
(b) D>0
(c) ewy okLrfod ,oa fHkUu&fHkUu gSA
(d) x  (– , )  (, ) es a f(x) > 0
(e) x  (, ) es a f(x) < 0

(ii) (a) a>0


(b) D=0
(c) ewy okLrfod ,oa leku gSA
(d) x  R – {} es a f(x) > 0

(iii) (a) a>0


(b) D<0
(c) ewy dkYifud gSA
(d) x  R ds fy, f(x) > 0

(iv) (a) a<0


(b) D>0
(c) ewy okLrfod ,oa fHkUu&fHkUu gSA
(d) x  (– , )  (, ) es a f(x) < 0
(e) x  (, ) es a f(x) > 0
(v) (a) a<0
(b) D=0
(c) ewy okLrfod ,oa leku gSA
(d) x  R – {} es a f(x) < 0

(vi) (a) a<0


(b) D<0
(c) ewy dkYifud gSA
(d) x  R ds fy, f(x) < 0

mnkjg.k # 10 : ;fn c < 0 ,oa ax 2 + bx + c = 0 dk dksbZ okLrfod ewy ugha gS rc fl) dhft, fd
(i) a–b+c<0 (ii) 9a + 3b + c < 0.
gy : c < 0 ,oa D < 0  f(x) = ax 2 + bx + c < 0  x  R
 f(– 1) = a – b + c < 0 ,oa f(3) = 9a + 3b + c < 0

mnkjg.k # 11 : f(x) = x 2 – 5x + 6 dk ifjlj Kkr dhft,A


b D
gy : x=– ij f(x) dk U;wure eku = – 4a
2a

5 D  25  24  1
 x= ij] – =–   =–
2 4a  4  4
 1 
f(x) dk vf/kdre eku   vr% ifjlj  ,   gS A
 4 

Corporate Office: CG Tower, A-46 & 52, IPIA, Near City Mall, Jhalawar Road, Kota (Raj.)-324005
Website : www.resonance.ac.in | E-mail : contact@resonance.ac.in
Toll Free : 1800 200 2244 | 1800 258 5555 | CIN: U80302RJ2007PTC024029 ADVQE - 6
f}?kkr lehdj.k
x2  x  4
mnkjg.k # 12 : ifjes; O;atd y = , x  R dk ifjlj Kkr dhft,A
x2  x  4
x2  x  4
gy : y=  (y – 1)x 2 + (y + 1) x + 4(y – 1) = 0 ........(i)
x2  x  4
fLFkfr- : ;fn y  1 gks] rks lehdj.k (i), x esa f}?kkr gSA
 x okLrfod gS ]
 D0  (y + 1)2 – 16(y – 1)2  0  (5y – 3) (3y – 5)  0

3 5
 y   ,  – {1}
5 3
fLFkfr- : ;fn y = 1 gks] rks lehdj.k (i) fuEu izdkj gks tkrh gS&
2x = 0  x = 0 tks la Hko gS A ¼D;ks a fd x  R)

3 5
 ifjlj   5 , 3 
 

x3
mnkjg.k # 13 : y = 2 , x  R dk ifjlj Kkr dhft,A
2x  3x  9

x3
gy : y= 2
2x  3x  9
 2yx2 + (3y – 1)x + 3(3y – 1) = 0 .......(i)
fLFkfr- : ;fn y  0 gks] rks lehdj.k (i), x esa f}?kkr gSA
 x okLrfod gS A
 D0
 (3y – 1)2 – 24y (3y – 1)  0
 (3y – 1) (21y + 1)  0

 1 1
y    ,  – {0}
 21 3 
fLFkfr- : ;fn y = 0 gks] rks lehdj.k (i) fuEu izdkj gks tkrh gS&
x = –2 tks la Hko gS A ¼D;ksafd x  R)
 1 1
 ifjlj y    21, 3 
 
vH;kl dk;Z :
(10) ;fn c > 0 gks ,oa ax 2 + 2bx + 3c = 0 dk dksbZ okLrfod ewy ugha gks] rks fl) dhft, fd
(i) 4a – 4b + 3c > 0 (ii) a + 6b + 9c > 0 (iii) a + 2b + 6c > 0

(a  b)2
(11) ;fn f(x) = (x – a) (x – b) gks] rks iznf'kZr dhft, fd f(x)  – .
4
(12) 'k' ds fdl U;w u re iw . kk± d eku ds fy, f}?kkr cgq in k – 1) x2 + 8x + k + 5 > 0  x  R gks xk \

x 2  34 x  71
(13) O;atd ] x  R dk ifjlj Kkr dhft,A
x 2  2x  7
mx 2  3 x  4
(14) O;atd ,  x  R ds lHkh okLrfod eku lEHko gks u s ds fy, 'm' fdl vUrjky es a fLFkr
 4x 2  3x  m
gksxk] Kkr dhft,A
Answers : (12) k=4 (13) (– , 5]  [9, ) (14) m  [1, 7]

Corporate Office: CG Tower, A-46 & 52, IPIA, Near City Mall, Jhalawar Road, Kota (Raj.)-324005
Website : www.resonance.ac.in | E-mail : contact@resonance.ac.in
Toll Free : 1800 200 2244 | 1800 258 5555 | CIN: U80302RJ2007PTC024029 ADVQE - 7
f}?kkr lehdj.k
11. ew yks a dh fLFkfr (Location of Roots) :
f (x) = ax² + bx + c, tgk¡ a > 0 ,oa a, b, c  R.

(i) (ii) (iii)

(i) f (x) = 0 ds nks u ks a ew y ,d fo'ks "k la [ ;k x 0 ls cM+ s gks u s ds fy, fuEu iz frcU/k gS &
b²  4ac  0 ,oa f (x 0) > 0 ,oa ( b/2a) > x 0
(ii) f (x) = 0 ds nks u ks a ew y ,d fo'ks "k la [ ;k x 0 ls Nks V s gks u s ds fy, fuEu iz frcU/k gS &
b²  4ac  0 ,oa f (x 0) > 0 ,oa ( b/2a) < x 0.
(iii) la[ ;k x 0 lehdj.k f (x) = 0 ds ewyksa ds e/; fLFkr gksus ds fy, izfrcU/k f (x 0) < 0 gSA

(iv) (v)

(iv) lehdj.k f (x) = 0 ds nksuksa ewy la[ ;k x 1 ,oa x 2 (x 1 < x 2) ds e/; fLFkr gksus ds fy, fuEu izfrcU/k gS&
b²  4ac  0 ,oa f (x 1) > 0 ,oa f (x 2) > 0 ,oa x 1 < ( b/2a) < x 2
(v) lehdj.k f (x) = 0 dk Bhd ,d ewy vUrjky (x 1, x 2) esa fLFkr vFkkZr~ x 1 < x < x 2 gksus ds fy, izfrcU/k
f (x 1). f (x 2) < 0 gS A
mnkgj.k # 14 : ekuk x 2 – (m – 3) x + m = 0 (m  R) ,d f}?kkr lehdj.k gS] rc m ds eku Kkr dhft, tcfd&
(a) nksuksa ewy 2 ls cMs+ gksA (b) nksuksa ewy /kukRed gksA
(c) ,d ewy /kukRed rFkk nwl jk _.kkRed gksA (d) ,d ewy 2 ls cM+k rFkk nwljk 1 ls NksVk gksA
(e) ewy ifjek.k esa cjkcj rFkk foijhr fpUg ds gksA
(f) nksuksa ewy vUrjky (1, 2) esa fLFkr gksA

gy : (a)

izfrcU/k - : D0  (m – 3)2 – 4m  0  m 2 – 10m + 9  0


 (m – 1) (m – 9)  0
 m  (– , 1]  [9, ) ......(i)
izfrcU/k - : f(2) > 0  4 – (m – 3)2 + m > 0 m < 10 ......(ii)
b m3
izfrcU/k - : – >2  2  m>7 .....(iii)
2a 2
loZfu"B ls m  [9, 10)

(b)

izfrcU/k -  D0  m  (– , 1] [9, )


izfrcU/k -  f(0) > 0  m>0
b m3
izfrcU/k -   >0  >0  m>3
2a 2
(i), (ii) ,oa (iii) ds loZ fu"B ls m  [9, ) Ans.
Corporate Office: CG Tower, A-46 & 52, IPIA, Near City Mall, Jhalawar Road, Kota (Raj.)-324005
Website : www.resonance.ac.in | E-mail : contact@resonance.ac.in
Toll Free : 1800 200 2244 | 1800 258 5555 | CIN: U80302RJ2007PTC024029 ADVQE - 8
f}?kkr lehdj.k

(c)

izfrcU/k -  f(0) < 0  m < 0 Ans.

(d)

izfrcU/k -  f(1) < 0  4<0  m 


izfrcU/k -  f(2) < 0  m > 10
loZfu"B ls m  Ans.
(e) ewyksa dk ;ksx = 0  m=3
,oa f(0) < 0  m<0
 m  Ans.

(f)

izfrcU/k -  D  0  m  (– , 1]  [9, )


izfrcU/k -  f(1) > 0  1 – (m – 3) + m > 0  4 > 0 tks fd 'm' ds çR;s d okLrfod eku ds
fy, lR; gS vr% m  R
izfrcU/k -  f(2) > 0  m < 10
b m3
izfrcU/k - V 1<– <2  1< <2  5<m<7
2a 2
loZfu"B ls m  Ans.

mnkgj.k # 15 : lehdj.k (a – 2)x2 – 2ax + a = 0 ds nksuksa ewy vUrjky (– 2, 1) esa fLFkr gksus ds fy, 'a' ds lHkh eku Kkr
dhft,&
gy : fLFkfr -I : f(–2) > 0  4(a – 2) + 4a + a > 0
8
9a – 8 > 0  a>
9
f(1) > 0  a – 2 – 2a + a > 0
– 2 > 0 laHko ugha
 a 
fLFkfr-II : a–2<0  a<2
8
f(–2) < 0  a<
9
f(1) < 0  aR

b 4
–2<– <1  a<
2a 3
D0  a0

 8
mHk;fu"B a  0, 
 9

lEiw.kZ gy a  0, 8   {2}


 9

Corporate Office: CG Tower, A-46 & 52, IPIA, Near City Mall, Jhalawar Road, Kota (Raj.)-324005
Website : www.resonance.ac.in | E-mail : contact@resonance.ac.in
Toll Free : 1800 200 2244 | 1800 258 5555 | CIN: U80302RJ2007PTC024029 ADVQE - 9
f}?kkr lehdj.k
vH;kl dk;Z %
(15) ;fn f}?kkr lehdj.k x2 – 2(a – 1)x + a – 1 = 0 (a  R) gks] rks a dk eku Kkr dhft, ftlds fy,
(a) nksuksa ewy /kukRed gksA (b) nksuksa ewy _.kkRed gksA
(c) nksuksa ewy foifjr fpUg ds gksA (d) nksuksa ewy 1 ls cM+s gksA
(e) nksuksa ewy 1 ls NksVs gksA
(f) ,d ewy 1 ls NksVk rFkk nwl jk ewy 1 ls cM+k gksA
(16) lehdj.k 4x 2 – 20px + (25p2 + 15p – 66) = 0 ds nksuksa ewy 2 ls NksVs gksus ds fy, 'p' ds eku Kkr dhft,A
(17)  ds os eku eku Kkr dhft, ftuds fy, 6, lehdj.k x 2 + 2( – 3)x + 9 = 0 ds ew yks a ds e/; fLFkr
gksxkA
(18) ;fn x2 – 2(a – 1)x + a – 1 = 0 (a  R) f}?kkr lehdj.k gks] rks 'a' dk eku Kkr dhft, ftlds fy,
(i) Bhd ,d ewy (0, 1) esa gksA (ii) nksuksa ewy (0, 1) esa gksA
(iii) de ls de ,d ewy (0, 1) esa gksA
(iv) ,d ewy 1 ls cM+k rFkk nwl jk 0 ls NksVk gksA
(19) f}?kkr O;atd ax 2 + (a – 2) x – 2 dk eku 'x' ds Bhd nks iw. kk±d ekuksa ds fy, _.kkRed gksus ds fy, 'a'
ds eku Kkr dhft,A
Answers : (15) (a) [2, ) (b)  (c) (– , 1) (d)  (e) (– , 1] (f) (2, )

 3
(16) (– , –1) (17)   ,  
 4
(18) (i) (– , 1)  (2, ) (ii)  (iii) ( – , 1)  (2, ) (iv) 
(19) [1, 2)

11. mHk;fu"B ew y (Common Roots) :


ekukfd nks f}?kkr lehdj.k a1 x 2 + b1 x + c1 = 0 ,oa a2 x 2 + b2 x + c2 = 0 gSaA
(i) ;fn nksuksa f}?kkr lehdj.kksa ds nksuksa ewy mHk;fu"B gks] rks nksuksa lehdj.ksa loZl e gksxh ,oa muds xq. kkad
a1 b c
lekuqikrh gksaxs vFkkZr~ = 1 = 1 .
a2 b2 c 2
(ii) ;fn dso y ,d ewy mHk;fu"B gks] rks mHk;fu"B ewy
c a  c2 a1 b c  b2 c1
= 1 2 = 1 2 gksxkA
a 1 b 2  a 2 b1
c1 a 2  c2 a1
2
vr% ,d ewy mHk;fu"B gksus ds fy, vko';d izfrcU/k c1 a 2  c2 a1  = a 1 b 2  a 2 b1  b1 c2  b2 c1  gksxkA
uks V : ;fn f(x) = 0 ,oa g(x) = 0 nks cgqinh; lehdj.ksa gS ftuds dqN ewy mHk;fu"B gS rks ;s mHk;fu"B ewy lehdj.k
h(x) = a f(x) + b g(x) = 0 ds Hkh ew y gks r s gS A
mnkjg.k # 16 : ;fn x 2 – ax + b = 0 rFkk x 2 – px + q = 0 nksuksa dk ,d ewy mHk;fu"B gS rFkk f}rh; lehdj.k esa nksuksa
ap
ewy leku gS rks iznf'kZr dhft, fd b + q = gksxkA
2
gy : fn, x, lehdj.k x 2 – ax + b = 0 ........ (i)
2
rFkk x – px + q = 0 ........ (ii) gS A
ekuk '' budk ,d mHk;fu"B ewy gSA rc lehdj.k (ii) ds ewy '' rFkk '' gksxsaA ekuk lehdj.k (i) dk nwl jk
ewy '' gSA bl izdkj lehdj.k (i) ds ewy ,  gS rFkk lehdj.k (ii) ds ewy , gSA
vc +=a ........ (iii)
 = b ........ (iv)
2 = p ........ (v)

Corporate Office: CG Tower, A-46 & 52, IPIA, Near City Mall, Jhalawar Road, Kota (Raj.)-324005
Website : www.resonance.ac.in | E-mail : contact@resonance.ac.in
Toll Free : 1800 200 2244 | 1800 258 5555 | CIN: U80302RJ2007PTC024029 ADVQE - 10
f}?kkr lehdj.k
2 = q ........ (vi)
2
cka; h vksj (L.H.S). = b + q =  +  = ( + ) ........ (vii)
ap (  ) 2
,oa nka; h vksj (R.H.S.) = = =  ( + ) ........ (viii)
2 2
(vii) ,oa (viii) ls ] cka; h vksj = nka; h vksj

mnkgj.k # 17 : ;fn a, b, c  R rFkk lehdj.kksa ax 2 + bx + c = 0 rFkk x 2 + 2x + 9 = 0 esa ,d ewy mHk;fu"B gS rks fl}
dhft, fd a : b : c = 1 : 2 : 9.
gy : nh x;h lehdj.ksa x 2 + 2x + 9 = 0 ........ (i)
2
rFkk ax + bx + c = 0 gSA ........ (ii)
Li"Vr% lehdj.k (i) ds ewy dkYifud gSA pw¡fd lehdj.k (i) ,oa (ii) dk ,d ewy mHk;fu"B gS] vr% mHk;fu"B
ewy dkYifud gksusa pkfg, rFkk bl izdkj nksuksa ewy mHk;fu"B gksxsa blfy, lehdj.k (i) ,oa (ii) loZle gSA
a b c
 = =  a:b:c=1:2:9
1 2 9
vH;kl dk;Z :
(20) ;fn lehdj.kksa ax 2 + bx + c = 0 rFkk x3 + x – 2 = 0 ds nks ewy mHk;fu"B gks rks iznf'kZr dhft, fd
2a = 2b = c.
a b c
(21) ;fn lehdj.kksa ax 2 + 2bx + c = 0 rFkk a1x 2 + 2b1x + c1 = 0 esa ,d ewy mHk;fu"B gks vkSj , ,
a1 b1 c1
lekUrj Js<+h esa gks] rks iznf'kZr fdft, fd a1, b1, c1 xq. kksÙkj Js<+h esa gSaA

12. cgq i nks a ds vkjs [ k (Graphs of Polynomials) :


y = anxn + ............ + a1x + a0. fcUnq tgk¡ y1 = 0 ] ifjofrZr fcUnq] tgk¡ vkjs[k dks [khpus ij ØkfUrd gSA

mnkjg.k # 18 : y = 2x3 – 15x2 + 36x + 1 dk vkjs[k [khpha,A


gy : y = 6x2 – 30x + 36 = 6(x – 3) (x – 2)

x 2 3  –

y 29 28  –

mnkjg.k # 19 : y = 3x4 + 4x3 + 3 dk vkjs[k [khpha,A


gy : y = –12x3 + 12x
y = –12x2 (x – 1)

x 0 1  –

y 3 4 – –

Corporate Office: CG Tower, A-46 & 52, IPIA, Near City Mall, Jhalawar Road, Kota (Raj.)-324005
Website : www.resonance.ac.in | E-mail : contact@resonance.ac.in
Toll Free : 1800 200 2244 | 1800 258 5555 | CIN: U80302RJ2007PTC024029 ADVQE - 11
Quadratic Equations

 Marked Questions may have for Revision Questions.


 fpfUgr iz ' u nks g jkus ;ks X; iz ' u gS A
PART - I : SUBJECTIVE QUESTIONS
Hkkx - I : fo"k;kRed iz'u ¼SUBJECTIVE QUESTIONS½
Section (A) : Relation between the roots and coefficients ; Quadratic Equation
[k.M (A) : ewyksa vkSj xq.kkadksa ds e/; lEcU/k vkSj f}?kkr lehdj.k
A-1. For what value of 'a', the equation (a2 – a – 2)x 2 + (a2 – 4)x + (a2 – 3a + 2) = 0, will have more than two
solutions ? Does there exist a real value of 'x' for which the above equation will be an identity in 'a' ?
'a' ds fdl eku ds fy, lehdj.k (a2 – a – 2)x 2 + (a2 – 4)x + (a2 – 3a + 2) = 0 ds nks ls vf/kd gy gks a xs \ D;k
'x' dk ,s l k dks bZ okLrfod eku fo|eku gS ftlds fy, nh xbZ lehdj.k 'a' es a ,d loZ l fedk gks \
Ans. a = 2; No real value of x. (a = 2; x ds fdlh Hkh okLrfod eku ds fy, ugha)

A-2. If  and  are the roots of the equation 2x 2 + 3x + 4 = 0, then find the values of
;fn lehdj.k 2x 2 + 3x + 4 = 0 ds ewy  ,oa  gksa] rks fuEu ds eku Kkr dhft,&
 
(i) 2 +  2 (ii) +
 

7 7
Ans. (i) – (ii) –
4 8

A-3. If  and  are the roots of the equation ax 2 + bx + c = 0, then find the equation whose roots are given
by
;fn f}?kkr lehdj.k ax 2 + bx + c = 0 ds ewy ,  gksa] rks og f}?kkr lehdj.k Kkr dhft, ftuds ewy fuEu gS&
1 1
(i) + ,+ (ii) 2 + 2,  2 + 2
 
Ans. (i) ac x 2 + b(a + c) x + (a + c) 2 = 0 (ii) a2 x 2 + (2ac – 4a2 – b2) x + 2b2 + (c – 2a)2 = 0

 
A-4. If  but 2 = 5 – 3,  2 = 5 – 3, then find the equation whose roots are and .
 

 
;fn  ysfdu 2 = 5 – 3,  2 = 5 – 3 gks] rks og lehdj.k Kkr dhft, ftlds ewy  ,oa  gksA
Ans. 3x 2 – 19x + 3 = 0.

A-5. In copying a quadratic equation of the form x 2 + px + q = 0, the coefficient of x was wrongly written
as – 10 in place of – 11 and the roots were found to be 4 and 6. Find the roots of the correct equation.
x 2 + px + q = 0 :i dh ,d f}?kkr lehdj.k dks fy[krs le; x dk xq. kka d xyrh ls –11 dh txg –10 fy[kus ij
ewy 4 ,oa 6 izkIr gksrs gks] rks lgh lehdj.k ds ewy Kkr dhft,A
Ans. 8, 3
3  5 1
A-6. (i) Find the value of the expression 2x 3 + 2x 2 – 7x + 72 when x = .
2
3  5 1
;fn x = gks] rks O;atd 2x 3 + 2x 2 – 7x + 72 dk eku Kkr dhft,A
2
1  15
(ii) Find the value of the expression 2x 3 + 2x 2 – 7x + 72 when x =
2
Corporate Office: CG Tower, A-46 & 52, IPIA, Near City Mall, Jhalawar Road, Kota (Raj.)-324005
Website : www.resonance.ac.in | E-mail : contact@resonance.ac.in
Toll Free : 1800 200 2244 | 1800 258 5555 | CIN: U80302RJ2007PTC024029 ADVQE - 12
Quadratic Equations
1  15
;fn x = gks] rks O;atd 2x 3 + 2x 2 – 7x + 72 dk eku Kkr dhft;sA
2
(iii) Solve the following equation 22x + 2x+2 – 32 = 0
fuEu lehdj.kksa dks gy dhft, 22x + 2x+2 – 32 = 0
Ans. (i) 4 (ii) 72 (iii) 2

A-7. Let a, b, c be real numbers with a  0 and let ,  be the roots of the equation ax 2 + bx + c = 0. Express
the roots of a3x 2 + abcx + c3 = 0 in terms of ,  [IIT-JEE-2002, Main., (4, 0)/100]
ekuk a, b, c okLrfod la[ ;k,¡ gS tcfd a  0 rFkk ekuk lehdj.k ax 2 + bx + c = 0 ds ewy ,  gSA
lehdj.k a3x 2 + abcx + c3 = 0 ds ewyksa dks , ds inksa esa O;Dr dhft,A
Ans.  = 2 and (vkS j )  =  2 or (;k)  =  2 and (vkS j ) = 2

A-8. If ,  are roots of x 2 – px + q = 0 and  – 2,  + 2 are roots of x 2 – px + r = 0, then prove that


16q + (r + 4 – q) 2 = 4p2.
;fn ,  lehdj.k x 2 – px + q = 0 ds ewy gS rFkk  – 2,  + 2 lehdj.k x 2 – px + r = 0 ds ewy gS rc fl)
dhft, fd 16q + (r + 4 – q)2 = 4p2.

A-9. If one root of the equation ax 2 + bx + c = 0 is equal to nth power of the other root, show that
(acn)1/(n + 1) + (anc)1/(n + 1) + b = 0.
;fn lehdj.k ax 2 + bx + c = 0 dk ,d ewy nwl js ewy dh noha ?kkr ds cjkcj gks] rks iznf'kZr dhft, fd
(acn)1/(n + 1) + (anc)1/(n + 1) + b = 0.

A-10. If the sum of the roots of quadratic equation (a + 1)x2 + (2a + 3)x + (3a + 4) = 0 is –1, then find the product
of the roots.
;fn f}?kkr lehdj.k (a + 1)x2 + (2a + 3)x + (3a + 4) = 0 ds ewyksa dk ;ksxQy –1 gS rc ewykas dk xq. kuQy Kkr
dhft,A
Ans. 2

Section (B) : Relation between roots and coefficients ; Higher Degree Equations
[k.M (B) : ewyksa vkSj xq.kkadksa ds e/; lEcU/k ; mPp ?kkr lehdj.ksa
B-1. If  and  be two real roots of the equation x 3 + px 2 + qx + r = 0 satisfying the relation  + 1 = 0, then
prove that r2 + pr + q + 1 = 0.
;fn lehdj.k x3 +px 2 + qx + r = 0 ds nks okLrfod ewy ,  bl izdkj gS fd lEcU/k  + 1 = 0 dks lUrq"V djrs
gS] rks fl) dhft, fd r2 + pr + q + 1 = 0.

B-2. If , ,  are the roots of the equation x 3 + px 2 + qx + r = 0, then find the value of
 1   1   1 
           .
        
 1   1   1 
;fn lehdj.k x 3 + px 2 + qx + r = 0 ds ewy , ,  gks] rks             dk eku Kkr dhft,A
        
(r  1)3
Ans. –
r2

B-3. (i) Solve the equation 24x 3 – 14x 2 – 63x +  = 0, one root being double of another. Hence find the
value(s) of .
lehdj.k 24x 3 – 14x 2 – 63x +  = 0 dks gy dhft, tcfd ,d ewy nwljs ewy dk nqxquk gS Qyr% dk@ds
eku Kkr dhft,A
(ii) Solve the equation 18x 3 + 81x 2 + x + 60 = 0, one root being half the sum of the other two.
Hence find the value of 
lehdj.k 18x 3 + 81x 2 + x + 60 = 0 dks gy dhft, tcfd ,d ewy] vU; nks ewykas ds ;ksxQy dk vk/kk gS
vr%  dk eku Kkr dhft,A
Corporate Office: CG Tower, A-46 & 52, IPIA, Near City Mall, Jhalawar Road, Kota (Raj.)-324005
Website : www.resonance.ac.in | E-mail : contact@resonance.ac.in
Toll Free : 1800 200 2244 | 1800 258 5555 | CIN: U80302RJ2007PTC024029 ADVQE - 13
Quadratic Equations
3 3 5 1 25
Ans. (i) roots are ewy gS , , ,  = 45 or ,  1, ,  = –25 .
4 2 3 2 12

4 3 5
(ii) roots are ewy gS , , ,  = 121
3 2 3

B-4. If , ,  are roots of equation x 3 – 6x 2 + 10x – 3 = 0, then find equation with roots 2 + 1, 2 + 1,
2 + 1.
;fn , ,  lehdj.k x 3 – 6x 2 + 10x – 3 = 0 ds ew y gS rc lehdj.k Kkr dhft, ftlds ew y
2 + 1, 2 + 1 vkS j 2 + 1 gS A
Ans. x 3 – 15x 2 + 67x – 77 = 0.
 
B-5. If ,  and  are roots of 2x 3 + x 2 – 7 = 0, then find the value of   .
, ,  
 
 
;fn lehdj.k 2x 3 + x 2 – 7 = 0 ds ewy , ,  gks] rks     dk
, ,  
  eku Kkr dhft,A

Ans. –3

Section (C) : Nature of Roots


[k.M (C) : ewykas dh çd`fr
C-1. If 2 + i 3 is a root of the equation x 2 + px + q = 0, where p, q  R, then find the ordered pair (p, q).

;fn lehdj.k x 2 + px + q = 0 tgk¡ p, q  R dk ,d ewy 2 + i 3 gks] rks Øfer ;qXe (p, q) Kkr dhft;sA
Ans. (– 4, 7)

9
C-2. If one root of equation ( – m) x 2 + x + 1 = 0 be double of the other and if  be real, show that m  .
8
;fn lehdj.k ( – m) x 2 + x + 1 = 0 dk ,d ewy nwl js ewy dk nqxquk gks vkSj  okLrfod gks] rks iznf'kZr dhft,
9
fd m  .
8
C-3. If the roots of the equation x 2 – 2cx + ab = 0 are real and unequal, then prove that the roots of
x 2 – 2(a + b) x + a2 + b2 + 2c2 = 0 will be imaginary.
;fn lehdj.k x 2 – 2cx + ab = 0 ds ew y okLrfod vkSj vleku gks] rks fl) dhft, fd lehdj.k
x 2 – 2(a + b) x + a2 + b2 + 2c2 = 0 ds ew y dkYifud gks a xs A

C-4. For what values of k the expression kx 2 + (k + 1)x + 2 will be a perfect square of a linear polynomial.
'k' ds fdu ekuks a ds fy, O;a td kx 2 + (k + 1)x + 2 ,d jS f[kd cgq in dk iw . kZ oxZ gks xk \
Ans. 3±2 2

C-5. Show that if roots of equation (a 2 – bc) x 2 + 2(b2 – ac) x + c 2 – ab = 0 are equal then either
b = 0 or a3 + b3 + c3 = 3abc
;fn lehdj.k (a2 – bc) x 2 + 2(b2 – ac) x + c2 – ab = 0 ds ewy leku gks] rks iznf'kZr dhft, fd ;k rks
b = 0 ;k a3 + b3 + c3 = 3abc gks xkA
1 1 1
C-6. If a, b, c  R, then prove that the roots of the equation + + = 0 are always real and
xa xb xc
cannot have roots if a = b = c.
1 1 1
;fn a, b, c  R, rks fl) dhft, fd lehdj.k + + = 0 ds ewy lnSo okLrfod gksxsa rFkk dksbZ
xa xb xc
ewy ugha gks ldrk ;fn a = b = c gksA
Corporate Office: CG Tower, A-46 & 52, IPIA, Near City Mall, Jhalawar Road, Kota (Raj.)-324005
Website : www.resonance.ac.in | E-mail : contact@resonance.ac.in
Toll Free : 1800 200 2244 | 1800 258 5555 | CIN: U80302RJ2007PTC024029 ADVQE - 14
Quadratic Equations

1 1 1
C-7. If the roots of the equation + = are equal in magnitude but opposite in sign, show that
( x  p) ( x  q) r
p + q = 2 r & that the product of the roots is equal to (1/2) (p² + q²).
1 1 1
;fn lehdj.k ( x  p) + ( x  q) = ds ewy ifjek.k esa cjkcj rFkk foijhr fpUg ds gks] rks iznf'kZr dhft,
r
fd p + q = 2 r ,oa ewyksa dk xq. kuQy (1/2) (p² + q²) gSA

C-8. (i) If – 2 + i,  R – {0} is a root of x3 + 63x +  = 0,  R then find roots of equation.
;fn – 2 + i,  R – {0} lehdj.k x3 + 63x +  = 0,  R dk ,d ewy gS rc lehdj.k ds ewy Kkr dhft,A
1
(ii) If + i,  R – {0} is a root of 2x3 + bx2 + 3x + 1 = 0, b  R, then find the value(s) of b.
2
1
;fn + i,  R – {0} lehdj.k 2x3 + bx2 + 3x + 1 = 0, b  R ds ewy gS rc b dk@ds eku Kkr dhft,A
2
Ans. (i) 4, – 2 ± i 5 3 (ii) 3 or 4

C-9. Solve the equation x 4 + 4x 3 + 5x 2 + 2x – 2 = 0, one root being – 1 + 1 .


lehdj.k x 4 + 4x 3 + 5x 2 + 2x – 2 = 0 dks gy dhft, tcfd bldk ,d ewy – 1 +  1 gksA
Ans. –1± 2,–1± 1

C-10. Draw graph of y = 12x3 – 4x2 – 3x + 1. Hence find number of positive zeroes.
y = 12x3 – 4x2 – 3x + 1 dk vkjs[k [khpha,A Qyr% /kukRed 'kwU;ksa dh la[;k Kkr dhft,A

Ans. . Two positive roots. nks /kukRed ewy gSA

Section (D) : Range of quadratic expression and sign of quadratic expression


[k.M (D) : f}?kkr O;atd dk ifjlj vkSj f}?kkr O;atd dk fpUg
D-1. Draw the graph of the following expressions : fuEufyf[kr f}?kkr O;a tdks a ds vkys [ k cukb, &
(i) y = x 2 + 4x + 3 (ii) y = 9x 2 + 6x + 1 (iii) y = – 2x 2 + x – 1

Ans./Sol. (i) (ii) (iii)

D-2. Find the range of following quadratic expressions :


fuEufyf[kr f}?kkr O;atdks ds ifjlj Kkr dhft, &
(i) f(x) = –x 2 + 2x + 3  xR Ans. (–, 4]
(ii) f(x) = x 2 – 2x + 3  x  [0, 3] Ans. [2, 6]
(iii) f(x) = x 2 – 4x + 6  x  (0, 1] Ans. [3, 6)

Corporate Office: CG Tower, A-46 & 52, IPIA, Near City Mall, Jhalawar Road, Kota (Raj.)-324005
Website : www.resonance.ac.in | E-mail : contact@resonance.ac.in
Toll Free : 1800 200 2244 | 1800 258 5555 | CIN: U80302RJ2007PTC024029 ADVQE - 15
Quadratic Equations
D-3. If x be real, then find the range of the following rational expressions :
;fn 'x' okLrfod gks] rks fuEufyf[kr ifjes; O;atdks ds ifjlj Kkr dhft,&
x2  x  1 1 3
(i) y= 2
Ans. 2, 2
x 1  

x 2  2x  9  4 
(ii)  y = 2 Ans.   , 5   (1, )
x – 2x  9  

kx 2  2(k  1)x  (9k  4)


D-4. Find the range of values of k, such that f(x) = is always negative.
x 2  8x  17

kx 2  2(k  1)x  (9k  4)


k ds ekuksa dk ifjlj Kkr dhft,] ;fn f(x) = ges'kk _.kkRed jgrk gSA
x 2  8x  17

 1
Ans.  ,  2 
 

D-5. x 2 + (a  b) x + (1  a  b) = 0, a, b  R. Find the condition on ' a ' for which


(i) Both roots of the equation are real and unequal  b  R .
(ii) Roots are imaginary  b  R
x 2 + (a  b) x + (1  a  b) = 0, a, b  R ' a ' ij izzfrcU/k Kkr dht, fd tcfd
(i) lehdj.k ds nksuksa ewy okLrfod vkSj vleku gS  b  R .
(ii) ewy dkYifud gS  b  R
Ans. (i) a > 1 (ii) a .

Section (E) : Location of Roots


[k.M (E) : ewyksa dh fLFkfr
E-1. If both roots of the equation x 2 – 6ax + 2 – 2a + 9a2 = 0 exceed 3, then show that a > 11/9.
;fn lehdj.k x 2 – 6ax + 2 – 2a + 9a2 = 0 ds nksuksa ewy 3 ls cM+s gks] rks iznf'kZr dhft, fd a > 11/9

E-2. Find all the values of 'K' for which one root of the equation x²  (K + 1) x + K² + K  8 = 0, exceeds 2 &
the other root is smaller than 2.
lehdj.k x²  (K + 1) x + K² + K  8 = 0 dk ,d ewy 2 ls cM+k ,oa nwl jk ewy 2 ls NksVk gksus ds fy, 'K' ds eku
Kkr dhft,A
Ans. K  ( 2, 3)

E-3. Find all the real values of 'a', so that the roots of the equation
(a2 – a + 2) x 2 + 2(a – 3) x + 9 (a4 – 16) = 0 are of opposite sign.
lehdj.k (a2 – a + 2) x 2 + 2(a – 3) x + 9 (a4 – 16) = 0 ds ewy foijhr fpUg ds gksus ds fy, 'a' ds lHkh okLrfod
eku Kkr dhft, A
Ans. a  (–2, 2)

E-4. Find all the values of 'a', so that exactly one root of the equation x 2 – 2ax + a2 – 1 = 0, lies between the
numbers 2 and 4, and no root of the equation is either equal to 2 or equal to 4.
lehdj.k x 2 – 2ax + a2 – 1 = 0 dk Bhd ,d ewy la[ ;kvksa 2 vkSj 4 ds e/; fLFkr gks rFkk lehdj.k dk
dksbZ Hkh ewy u rks 2 ds cjkcj gks vkSj u gh 4 ds cjkcj gks] rks 'a' ds lHkh eku Kkr dhft,A
Ans. a  (1, 5) – {3}

E-5. If  &  are the two distinct roots of x² + 2 (K  3) x + 9 = 0, then find the values of K such that
,   ( 6, 1).

Corporate Office: CG Tower, A-46 & 52, IPIA, Near City Mall, Jhalawar Road, Kota (Raj.)-324005
Website : www.resonance.ac.in | E-mail : contact@resonance.ac.in
Toll Free : 1800 200 2244 | 1800 258 5555 | CIN: U80302RJ2007PTC024029 ADVQE - 16
Quadratic Equations
;fn x² + 2 (K  3) x + 9 = 0 ds ewy  ,oa  gks] rks 'K' dk eku Kkr dhft, tcfd ,   ( 6, 1).
Ans. 6 < K < 6.75
Sol. x 2 + 2(k – 3) x + 9 = 0 ......(i)
Roots , of equation (i) are distinct & lies between –6 and 1
lehdj.k (i) ds ewy ,  fHkUu&fHkUu gS ftuds eku vUrjky (–6, 1) esa gSA
D>0  4(K – 3)2 – 36 > 0  k(k – 6) > 0
 k (– , 0) (6, ) ......(ii)
f(1) > 0 1 + 2 (k – 3)+ 9 > 0
 2k + 4 > 0
 k (–2, ) ......(iii)
f(–6) > 0 36 – 12 (k – 3) + 9 > 0
 27 
 4k – 27 < 0  k   – , 4  ......(iv)
 
b –2(K – 3)
–6 < – < 1 – 6 < < 1  –1 < k – 3 < 6  2 < k < 9 ......(v)
2a 2
 27 
(ii)  (iii) (iv) (v) ls  k   6, .
 4 

Section (F) : Common Roots & Graphs of Polynomials


[k.M (F) : mHk;fu"B ewy ,oa cgqinksa ds vkjs[ k
F-1. If one of the roots of the equation ax 2 + b x + c = 0 be reciprocal of one of the roots of
a1 x 2 + b1 x + c1 = 0, then prove that (a a1  c c1)2 = (b c1  a b1) (b1c  a1b).
;fn lehdj.k a x 2 + b x + c = 0 dk dksbZ ,d ewy lehdj.k a1 x 2 + b1 x + c1 = 0 ds ewyksa esa ls ,d dk O;qRØe
gS rks iznf'kZr dhft, fd (a a1  c c1)2 = (b c1  a b1) (b1c  a1b).
1
Sol. If  is one of the root of a1x2 + b1x + c1 = 0. Then will be a root of ax2 + bx + c = 0  c2 + b + a = 0

& a12 + b1 + c1 = 0 have one common root.
 applying the condition for one common root we get (aa1 – cc1)2 = (bc1 – ab1) (b1c – a1b)
1
Hindi ;fn lehdj.k a1x2 + b1x + c1 = 0 dk ,d ewy '' gks] rks lehdj.k ax2 + bx + c = 0 dk ,d ewy gksxkA

 c2 + b + a = 0 vkSj a12 + b1 + c1 = 0 esa ,d ewy mHk;fu"B gSA
 ,d ewy mHk;fu"B gksus dk çfrcU/k yxkusa ij (aa1 – cc1)2 = (bc1 – ab1) (b1c – a1b)

F-2. Find the value of 'a' so that x 2 – 11 x + a = 0 and x 2 – 14x + 2a = 0 have a common root.
x 2 – 11 x + a = 0 vkS j x 2 – 14x + 2a = 0 dk ,d ew y mHk;fu"B gks u s ds fy, 'a' ds eku Kkr dhft,A
Ans. a = 0, 24
Sol. Given equation are x 2 – 11x + a = 0 .........(i) x 2 – 14x + 2a = 0 .........(ii)
2
Multiplying equation (i) by 2 and then subtracting, we get x – 8x = 0  x = 0, 8
If x = 0, a = 0 If x = 8, a = 24
2
Hindi. nh x;h lehdj.ks a gS & x – 11x + a = 0 .........(i) x 2 – 14x + 2a = 0 .........(ii)
lehdj.k (i) dks 2 ls xq. kk djdsa blesa ls lehdj.k (ii) dks ?kVkusa ij x 2 – 8x = 0  x = 0, 8
;fn x = 0 rks a=0 ;fn x = 8, a = 24

F-3. If ax 2 + bx + c = 0 and bx 2 + cx + a = 0 have a common root and a, b, c are non-zero real numbers, then

a3  b3  c 3
find the value of .
abc

Corporate Office: CG Tower, A-46 & 52, IPIA, Near City Mall, Jhalawar Road, Kota (Raj.)-324005
Website : www.resonance.ac.in | E-mail : contact@resonance.ac.in
Toll Free : 1800 200 2244 | 1800 258 5555 | CIN: U80302RJ2007PTC024029 ADVQE - 17
Quadratic Equations
;fn ax 2 + bx + c = 0 vkSj bx 2 + cx + a = 0 dk ,d ewy mHk;fu"B gks vkSj a, b, c v'kwU; okLrfod la[ ;k,¡ gks]
a3  b3  c 3
rks dk eku Kkr dhft,A
abc
Ans. 3

F-4. If x 2 + px + q = 0 and x 2 + qx + p = 0, (p  q) have a common root, show that 1 + p + q = 0 ; show that


their other roots are the roots of the equation x 2 + x + pq = 0.
;fn x 2 + px + q = 0 vkSj x 2 + qx + p = 0, (p  q) dk ,d ewy mHk;fu"B gks] rks iznf'kZr dhft, fd
1 + p + q = 0 rFkk ;g Hkh iz n f'kZ r dhft, fd 'ks "k ew y lehdj.k x 2 + x + pq = 0 ds ew y gS a A

F-5. Draw the graphs of following :


fuEu ds vkjs[ k [khph,saA
(i) y = 2x3 + 9x2 – 24x + 15 (ii) y = – 3x4 + 4x3 + 12x2 – 2

Ans. (i) (ii)

PART - II : ONLY ONE OPTION CORRECT TYPE


Hkkx - II : dsoy ,d lgh fodYi çdkj (ONLY ONE OPTION CORRECT TYPE)
Section (A) : Relation between the roots and coefficients quadratic equation
[k.M (A) : ewyksa vkSj xq.kkadksa ds e/; lEcU/k vkSj f}?kkr lehdj.k

A-1. The roots of the equation (b – c) x 2 + (c – a) x + (a – b) = 0 are


lehdj.k (b – c) x 2 + (c – a) x + (a – b) = 0 ds ewy gaS&
ca ab bc ca
(A) ,1 (B*) ,1 (C) ,1 (D) ,1
bc bc ab ab

A-2. If ,  are the roots of quadratic equation x 2 + p x + q = 0 and ,  are the roots of x 2 + p x – r = 0,
then () . () is equal to :
;fn f}?kkr lehdj.k x 2 + p x + q = 0 ds ewy ,  gks vkSj lehdj.k x 2 + p x – r = 0 ds ewy ,  gks] rks
() . () dk eku gS &
(A) q + r (B) q – r (C*) – (q + r) (D) – (p + q + r)

Corporate Office: CG Tower, A-46 & 52, IPIA, Near City Mall, Jhalawar Road, Kota (Raj.)-324005
Website : www.resonance.ac.in | E-mail : contact@resonance.ac.in
Toll Free : 1800 200 2244 | 1800 258 5555 | CIN: U80302RJ2007PTC024029 ADVQE - 18
Quadratic Equations
A-3. Two real numbers  &  are such that  +  = 3,    = 4, then  &  are the roots of the quadratic
equation:
(A*) 4x 2  12x  7 = 0 (B) 4x 2  12x + 7 = 0 (C) 4x 2  12x + 25 = 0 (D) none of these
nks okLrfod la[ ;k,¡  ,oa  bl izdkj gS fd  +  = 3 ,oa    = 4 gks] rks og f}?kkr lehdj.k ftlds ewy 
,oa  gS] gksxh&
(A) 4x 2  12x  7 = 0 (B) 4x 2  12x + 7 = 0
2
(C) 4x  12x + 25 = 0 (D) bues a ls dks bZ ugha A

A-4. For the equation 3x 2 + px + 3 = 0, p > 0 if one of the roots is square of the other, then p is equal to:
lehdj.k 3 x 2 + px + 3 = 0, p > 0 ds fy, ;fn ,d ewy nwl js dk oxZ gks] rks p dk eku gS&
(A) 1/3 (B) 1 (C*) 3 (D) 2/3

A-5. Consider the following statements :


S1 : If the roots of x2 – bx + c = 0 are two consecutive integers, then value of b2 – 4c is equal to 1.
S2 : If  are roots of x2 – x + 3 = 0 then value of 4 is equal 7.
S3 : If  are the roots of x3 – 7x2 + 16 x – 12 = 0 then value of 2 + 2 +  is equal to 17.
State, in order, whether S1, S2, S3 are true or false
fuEufyf[kr dFkuksa ij fopkj dhft, :
S1 : ;fn lehdj.k x2 – bx + c = 0 ds ewy nks Øekxr iw.kk±d gks] rks b2 – 4c dk eku 1 gSA
S2 : ;fn x2 – x + 3 = 0 ds ewy gks] rks 4 dk eku 7 gSA
S3 : ;fn x3 – 7x2 + 16 x – 12 = 0 ds ewy  gks] rks 2 + 2 +  dk eku 17 gSA
S1, S2, S3 ds lR; (T) ;k vlR; (F) gks u s dk lgh Øe gS &
(A*) TTT (B) FTF (C) TFT (D) FTT

Section (B) : Relation between roots and coefficients ; Higher Degree Equations
[k.M (B) : ewyksa vkSj xq.kkadksa ds e/; lEcU/k ; mPp ?kkr lehdj.ksa

B-1. If two roots of the equation x 3  px 2 + qx  r = 0, (r  0) are equal in magnitude but opposite in sign,
then:
(A) pr = q (B) qr = p (C*) pq = r (D) None of these
;fn lehdj.k x 3  px 2 + qx  r = 0, (r  0) ds nks ewy ekikad esa cjkcj ijUrq foijhr fpUg ds gks] rks &
(A) pr = q (B) qr = p (C) pq = r (D) buesa ls dksbZ ughA

1  1  1 
B-2. If , &  are the roots of the equation x 3  x  1 = 0 then, + + has the value equal to:
1  1  1 
(A) zero (B)  1 (C*)  7 (D) 1
1  1  1 
;fn lehdj.k x 3  x  1 = 0 ds ewy , ,oa  gks] rks + + dk eku gS&
1  1  1 
(A) 'kwU; (B)  1 (C)  7 (D) 1

B-3. Let , , be the roots of (x – a) (x – b) (x – c) = d, d  0, then the roots of the equation
(x – ) (x – ) (x – ) + d = 0 are :
;fn lehdj.k (x – a) (x – b) (x – c) = d, d  0, ds ewy  gks] rks lehdj.k
(x – ) (x – ) (x – ) + d = 0 ds ew y gS a &
(A) a + 1, b + 1, c + 1 (B*) a, b, c
a b c
(C) a – 1, b – 1, c – 1 (D) , ,
b c a

Corporate Office: CG Tower, A-46 & 52, IPIA, Near City Mall, Jhalawar Road, Kota (Raj.)-324005
Website : www.resonance.ac.in | E-mail : contact@resonance.ac.in
Toll Free : 1800 200 2244 | 1800 258 5555 | CIN: U80302RJ2007PTC024029 ADVQE - 19
Quadratic Equations
Section (C) : Nature of Roots
[k.M (C) : ewykas dh çd`fr

C-1. If a, b, c are integers and b2 = 4(ac + 5d2), d  N, then roots of the quadratic equation ax 2 + bx + c = 0
are
(A*) Irrational (B) Rational & different
(C) Complex conjugate (D) Rational & equal
;fn a, b, c iw. kk±d gks vkSj b2 = 4(ac + 5d2), d  N, rks f}?kkr lehdj.k ax 2 + bx + c = 0 ds ewy gSa &
(A) vifjes; (B) ifjes ; vkSj vleku (C) lfEeJ la ;q Xeh (D) ifjes ; vkSj leku

C-2. Let a, b and c be real numbers such that 4a + 2b + c = 0 and ab > 0. Then the equation
ax 2 + bx + c = 0 has
(A*) real roots (B) imaginary roots (C) exactly one root (D) none of these
;fn a, b ,oa c okLrfod la[ ;k,¡ bl izdkj gS fd 4a + 2b + c = 0 rFkk ab > 0 gks] rks lehdj.k
ax 2 + bx + c = 0
(A) ds nks u ks a ew y okLrfod gS A (B) ds nks u ks a ew y dkYifud gS A
(C) dk Bhd ,d ew y gS A (D) bues a ls dks bZ ugha A

C-3. Consider the equation x 2 + 2x – n = 0, where n  N and n  [5, 100]. Total number of different values of
'n' so that the given equation has integral roots, is
fdlh lehdj.k x 2 + 2x – n = 0, tgk¡ n  N vkSj n  [5, 100], ds ewy iw. kk±d gksus ds fy, n ds fHkUu&fHkUu ekuksa
dh dqy la[ ;k gS &
(A) 4 (B) 6 (C*) 8 (D) 3

Section (D) : Range of quadratic expression and sign of quadratic expression


[k.M (D) : f}?kkr O;atd dk ifjlj vkSj f}?kkr O;atd dk fpUg
D-1. If  &  ( < ) are the roots of the equation x 2 + bx + c = 0, where c < 0 < b, then
;fn lehdj.k x 2 + bx + c = 0 ¼tgk¡ c < 0 < b½ ds ewy  ,oa  ( < ) gks] rks&
(A) 0 <  <  (B*)  < 0 <  2< 2
(C) <  < 0 (D) < 0 < 2 <  2

D-2. W hich of the following graph represents the expression f(x) = a x 2 + b x + c (a  0) when
a > 0, b < 0 & c < 0 ?
fuEufyf[kr esa ls dkSu&lk vkys[ k O;atd f (x) = a x 2 + b x + c (a  0) dks iznf'kZr djrk gS tcfd a > 0, b < 0
,oa c < 0 gks &

(A) (B*) (C) (D)

D-3. The expression y = ax 2 + bx + c has always the same sign as of 'a' if :


(A) 4ac < b2 (B*) 4ac > b2 (C) 4ac = b2 (D) ac < b2
O;tad y = ax 2 + bx + c dk fpUg lnSo 'a' ds fpUg ds leku gksrk gS ;fn &
(A) 4ac < b2 (B*) 4ac > b2 (C) 4ac = b2 (D) ac < b2

D-4. The entire graph of the expression y = x 2 + kx – x + 9 is strictly above the x-axis if and only if
(A) k < 7 (B*) –5 < k < 7 (C) k > – 5 (D) none
2
O;atd y = x + kx – x + 9 dk lEiw. kZ vkys[ k x-v{k ls Åij gksxk ;fn vkSj dso y ;fn &
(A) k < 7 (B) –5 < k < 7 (C) k > – 5 (D) bues a ls dks bZ ugha A
Corporate Office: CG Tower, A-46 & 52, IPIA, Near City Mall, Jhalawar Road, Kota (Raj.)-324005
Website : www.resonance.ac.in | E-mail : contact@resonance.ac.in
Toll Free : 1800 200 2244 | 1800 258 5555 | CIN: U80302RJ2007PTC024029 ADVQE - 20
Quadratic Equations
D-5. If a, b  R, a  0 and the quadratic equation ax 2  bx + 1 = 0 has imaginary roots then a + b + 1 is:
(A*) positive (B) negative (C) zero (D) depends on the sign of b
;fn a, b  R, a  0 ,oa f}?kkr lehdj.k ax 2  bx + 1 = 0 ds ewy dkYifud gks] rks a + b + 1 gksxk&
(A) /kukRed (B) _.kkRed (C) 'kwU; (D) b ds fpUg ij fuHkZ j A

D-6. If a and b are the non-zero distinct roots of x 2 + ax + b = 0, then the least value of x 2 + ax + b is
;fn lehdj.k x 2 + ax + b = 0 ds v'kwU; fHkUu&fHkUu ewy 'a' ,oa 'b' gks] rks x 2 + ax + b dk U;wure eku gS&
3 9 9
(A) (B) (C*) – (D) 1
2 4 4

D-7. If y = – 2x 2 – 6x + 9, then
(A) maximum value of y is –11 and it occurs at x = 2
(B) minimum value of y is –11 and it occurs at x = 2
(C*) maximum value of y is 13.5 and it occurs at x = –1.5
(D) minimum value of y is 13.5 and it occurs at x = –1.5
;fn y = – 2x 2 – 6x + 9 gks] rks &
(A) y dk vf/kdre eku –11 gS vkS j ;g x = 2 ij iz kIr gks r k gS A
(B) y dk U;w u re eku –11 gS vkS j ;g x = 2 ij iz kIr gks r k gS A
(C) y dk vf/kdre eku 13.5 gS vkS j ;g x = –1.5 ij iz kIr gks r k gS A
(D) y dk U;w u re eku 13.5 gS vkS j ;g x = –1.5 ij iz kIr gks r k gS A

D-8. If f(x) = x 2 + 2bx + 2c2 and g(x) = – x 2 – 2cx + b2 are such that min f(x) > max g(x), then the relation
between b and c, is
(A) no relation (B) 0 < c < b/2 (C) c2 < 2b (D*) c2 > 2b2
;fn f(x) = x 2 + 2bx + 2c2 ,oa g(x) = – x 2 – 2cx + b2 bl izdkj gS fd min f(x) > max g(x) gS] rks b ,oa c esa
lEcU/k gS&
(A) dks bZ lEcU/k ugha (B) 0 < c < b/2 (C) c2 < 2b (D*) c2 > 2b2

Section (E) : Location of Roots


[k.M (E) : ewyksa dh fLFkfr

E-1. If b > a, then the equation (x  a) (x  b)  1 = 0, has:


(A) both roots in [a, b] (B) both roots in (a)
(C) both roots in [b) (D*) one root in (a) & other in (b, )
;fn b > a gks] rks lehdj.k (x  a) (x  b)  1 = 0
(A) ds nks u ks a ew y vUrjky [a, b] es a gS A (B) ds nks u ks a ew y vUrjky (a) es a gS A
(C) ds nks u ks a ew y vUrjky [b) es a gS A (D) dk ,d ew y vUrjky (a) es a ,oa nw l jk (b, ) es a gS A

E-2. If , are the roots of the quadratic equation x 2  2p (x  4)  15 = 0, then the set of values of 'p' for
which one root is less than 1 & the other root is greater than 2 is:
(A) (7/3, ) (B*) (, 7/3) (C) x  R (D) none
2
;fn f}?kkr lehdj.k x  2p (x  4)  15 = 0 ds ewy ,gks] rks buesa ls ,d ewy 1 ls NksVk vkSj nwl jk ewy 2
ls cM+k gksus ds fy, 'p' ds ekuksa dk leqPp; gS&
(A) (7/3, ) (B) (, 7/3) (C) x  R (D) bues a ls dks bZ ugha A

E-3. If ,  be the roots of 4x 2 – 16x +  = 0, where   R, such that 1 <  < 2 and 2 <  < 3, then the number
of integral solutions of  is
;fn lehdj.k 4x 2 – 16x +  = 0 tgk¡   R, ds ewy ,  bl izdkj gS fd 1 <  < 2 vkSj 2 <  < 3 gks] rks ''
ds iw. kk±d gyksa dh la[ ;k gS &
(A) 5 (B) 6 (C) 2 (D*) 3
Corporate Office: CG Tower, A-46 & 52, IPIA, Near City Mall, Jhalawar Road, Kota (Raj.)-324005
Website : www.resonance.ac.in | E-mail : contact@resonance.ac.in
Toll Free : 1800 200 2244 | 1800 258 5555 | CIN: U80302RJ2007PTC024029 ADVQE - 21
Quadratic Equations
Section (F) : Common Roots & Graphs of Polynomials
[k.M (F) : mHk;fu"B ewy ,oa cgqinksa ds vkjs[ k
F-1. If the equations k (6x 2 + 3) + rx + 2x 2 – 1 = 0 and 6k (2x 2 + 1) + px + 4x 2 – 2 = 0 have both roots
common, then the value of (2r – p) is
(A*) 0 (B) 1/2 (C) 1 (D) none of these
;fn lehdj.kksa k (6x 2 + 3) + rx + 2x 2 – 1 = 0 ,oa 6k (2x 2 + 1) + px + 4x 2 – 2 = 0 ds nks u ks a ew y mHk;fu"B gks ]
rks (2r – p) dk eku gS&
(A) 0 (B) 1/2 (C) 1 (D) bues a ls dks bZ ugha A
F-2. If 3x2 – 17x + 10 = 0 and x2 – 5x +  = 0 has a common root, then sum of all possible real values of  is
29 26 29
(A) 0 (B)  (C*) (D)
9 9 3
2 2
;fn 3x – 17x + 10 = 0 vkSj x – 5x +  = 0 dk ,d mHk;fu"B ewy gS rc  ds lHkh laHkkfor okLrfod ekuksa dk
;ksxQy gS
29 26 29
(A) 0 (B)  (C*) (D)
9 9 3

F-3. If a, b, p, q are nonzero real numbers, then two equations 2a 2 x 2  2 ab x + b 2 = 0 and


p2 x 2 + 2 pq x + q2 = 0 have :
(A*) no common root (B) one common root if 2 a2 + b2 = p2 + q2
(C) two common roots if 3 pq = 2 ab (D) two common roots if 3 qb = 2 ap
;fn a, b, p, q v'kwU; okLrfod la[ ;k,¡ gS] rks lehdj.kksa 2 a2 x 2  2 ab x + b2 = 0 ,oa p2 x 2 + 2 pq x + q2 = 0 ds
fy,
(A) dks bZ mHk;fu"B ew y ugha gS A (B) ,d ew y mHk;fu"B gS ] ;fn 2 a2 + b2 = p2 + q2
(C) nks ew y mHk;fu"B gS ] ;fn 3 pq = 2 ab (D) nks ew y mHk;fu"B gS ] ;fn 3 qb = 2 ap

x3 – 4x
F-4. The graphs of y = is
4

x3 – 4x
y= dk vkjs[ k gS&
4

(A) (B)

Corporate Office: CG Tower, A-46 & 52, IPIA, Near City Mall, Jhalawar Road, Kota (Raj.)-324005
Website : www.resonance.ac.in | E-mail : contact@resonance.ac.in
Toll Free : 1800 200 2244 | 1800 258 5555 | CIN: U80302RJ2007PTC024029 ADVQE - 22
Quadratic Equations

(C*) (D)

F-5. The graphs of y = x 4 – 2x 2 + 5 is


y = x 4 – 2x 2 + 5 dk vkjs [ k gS &

(A) (B)

(C) (D*)

PART - III : MATCH THE COLUMN


Hkkx - III : dkWye dks lqesfyr dhft, (MATCH THE COLUMN )

1. Column –  Column – 
(A) If  + 4 are two roots of x2 – 8 x + k = 0, (p) 4
then possible value of k is
1 1
(B) If ,  are roots of x2 + 2x – 4 = 0 and , are (q) 0
 

3
roots of x2 + qx + r = 0 then value of is
qr

Corporate Office: CG Tower, A-46 & 52, IPIA, Near City Mall, Jhalawar Road, Kota (Raj.)-324005
Website : www.resonance.ac.in | E-mail : contact@resonance.ac.in
Toll Free : 1800 200 2244 | 1800 258 5555 | CIN: U80302RJ2007PTC024029 ADVQE - 23
Quadratic Equations
(C) If ,  are roots of ax2 + c = 0, ac  0, then (r) 12
3 + 3 is equal to
(D) If roots of x2 – kx + 36 = 0 (s) 10
are Integers then number of values of k =
LrEHk –  LrEHk – 
2
(A) ;fn x – 8 x + k = 0 ds ewy ,  + 4 gks] rks 'k' dk lEHkkfor (p) 4
eku gS &
1 1
(B)_ ;fn ,  lehdj.k x2 + 2x – 4 = 0 ds ewy gS rFkk  ,  (q) 0

3
lehdj.k x2 + qx + r = 0 ds ewy gS rc q  r dk eku gS&

(C)_ ;fn lehdj.k ax2 + c = 0, ac  0 ds ewy ,  gS rc (r) 12


3 + 3 cjkcj gS&
(D) ;fn x2 – kx + 36 = 0 ds ewy iw.kk±d gS (s) 10
rc k ds ekuksa dh la[;k gS&
Ans. (A)  (r), (B)  (p), (C)  (q), (D)  (s)

2. If graph of the expression f(x) = ax2 + bx + c (a  0) are given in column-II, then Match the items in column-I with
in column-II (where D = b2 – 4ac)
;fn f}?kkr O;atd f(x) = ax2 + bx + c, (a  0) dk xzkQ LrEHk-II esa iznf'kZr gS rks LrEHk-I dk LrEHk-II ;s laHkkfor feyku djksA
(tgk¡ D = b2 – 4ac)
Column-I Column-II

abc
(A) 0 (p)
D

abc
(B) 0 (q)
D

(C) abc > 0 (r)

(D) abc < 0 (s)

Ans. (A  r); (B  p,q,s); (C  s); (D  p,q,r)

Corporate Office: CG Tower, A-46 & 52, IPIA, Near City Mall, Jhalawar Road, Kota (Raj.)-324005
Website : www.resonance.ac.in | E-mail : contact@resonance.ac.in
Toll Free : 1800 200 2244 | 1800 258 5555 | CIN: U80302RJ2007PTC024029 ADVQE - 24
Quadratic Equations
3. Let y = Q(x) = ax 2 + bx + c be a quadratic expression. Match the inequalities in Column-I with possible
graphs in Column-II.
ekuk y = Q(x) = ax2 + bx + c ,d f}?kkr O;atd gSA LrEHk - I eas vlfedkvksa dks LrEHk - II eas laHkkfor vkjs[k ls feykb;s
Column-I Column-II
LrEHk - II LrEHk - II

(A) Q(x) > 0,  x  (2, 7) (p)

(B) Q(x) > 0,  x  (– , 1) (q)

(C) Q(x) < 0,  x  (1, 6) (r)

(D) Q(x) < 0,  x  (– , – 1) (s)

(t)

Ans. (A) q, s, t (B) p, t (C) r (D) q, s.

Corporate Office: CG Tower, A-46 & 52, IPIA, Near City Mall, Jhalawar Road, Kota (Raj.)-324005
Website : www.resonance.ac.in | E-mail : contact@resonance.ac.in
Toll Free : 1800 200 2244 | 1800 258 5555 | CIN: U80302RJ2007PTC024029 ADVQE - 25
Quadratic Equation

 Marked Questions may have for Revision Questions.


 fpfUgr iz ' u nks g jkus ;ks X; iz ' u gS A

PART - I : ONLY ONE OPTION CORRECT TYPE


Hkkx-I : dsoy ,d lgh fodYi çdkj (ONLY ONE OPTION CORRECT TYPE)

1. Let a > 0, b > 0 & c > 0. Then both the roots of the equation ax 2 + bx + c = 0
(A) are real & negative (B*) have negative real parts
(C) are rational numbers (D) have positive real parts
;fn a > 0, b > 0 ,oa c > 0 gks] rks lehdj.k ax 2 + bx + c = 0 ds &
(A) nks u ks ew y okLrfod ,oa _.kkRed gS A (B) nks u ks a ew yks a ds okLrfod Hkkx _.kkRed gS A
(C) nks u ks ew y ifjes ; la [ ;k,¡ gS A (D) nks u ks a ew yks a ds okLrfod Hkkx /kukRed gS A

2. If the roots of the equation x 2 + 2ax + b = 0 are real and distinct and they differ by at most 2m, then b
lies in the interval
(A) (a2 – m 2, a2) (B*) [a2 – m 2, a2) (C) (a2, a2 + m 2) (D) none of these
2
;fn lehdj.k x + 2ax + b = 0 ds ewy okLrfod ,oa fHkUu&fHkUu gks rFkk mudk vUrj vf/kdre 2m gks] rks b fuEu
vUrjky esa fLFkr gS&
(A) (a2 – m 2, a2) (B) [a2 – m 2, a2) (C) (a2, a2 + m 2) (D) bues a ls dks bZ ugha

3. The set of possible values of  for which x2 – (2 – 5 + 5)x + (22 – 3 – 4) = 0 has roots, whose sum and
product are both less than 1, is
lehdj.k x 2 – (2 – 5 + 5)x + (22 – 3 – 4) = 0 ds ewyksa dk ;ksx ,oa xq. kuQy nksuksa 1 ls NksVs gksus ds fy, ''
ds laHko ekuksa dk leqPp; gS &
 5  5  5
(A)   1, 2  (B) (1, 4) (C) 1 , 2  (D*) 1, 2 
 

4. If p, q, r, s  R, then equaton (x 2 + px + 3q) (–x 2 + rx + q) (–x 2 + sx – 2q) = 0 has


(A) 6 real roots (B*) at least two real roots
(C) 2 real and 4 imaginary roots (D) 4 real and 2 imaginary roots
;fn p, q, r, s  R gks] rks lehdj.k (x 2 + px + 3q) (–x 2 + rx + q) (–x 2 + sx – 2q) = 0 ds fy, &
(A) 6 okLrfod ew y gS A (B) de ls de nks okLrfod ew y gS A
(C) 2 okLrfod vkS j 4 dkYifud ew y gS A (D) 4 okLrfod vkS j 2 dkYifud ew y gS A

5. If coefficients of biquadratic equation are all distinct and belong to the set {–9, – 5, 3, 4, 7}, then equation has
(A*) at least two real roots
(B) four real roots, two are conjugate surds and other two are also conjugate surds
(C) four imaginary roots
(D) None of these
;fn pkj ?kkr okyh lehdj.k ds xq.kkad] lHkh fofHkUu gS rFkk leqPp; {–9, – 5, 3, 4, 7} ds vo;o gS] rc lehdj.k j[krh
gS&
(A*) de ls de nks okLrfod ewy
(B) pkj okLrfod ewy] nks l;qXeh dj.kh ewy vkSj vU; nks Hkh la;qXeh dj.kh ewy
(C) pkj dkYifud ewy
(D) buesa ls dksbZ ugha

Corporate Office: CG Tower, A-46 & 52, IPIA, Near City Mall, Jhalawar Road, Kota (Raj.)-324005
Website : www.resonance.ac.in | E-mail : contact@resonance.ac.in
Toll Free : 1800 200 2244 | 1800 258 5555 | CIN: U80302RJ2007PTC024029
ADVQE - 26
Quadratic Equation
6. Let p, q, r, s  R, x 2 + px + q = 0, x 2 + rx + s = 0 such that 2 (q + s) = pr then
(A*) at least one of the equation have real roots.
(B) either both equations have imaginary roots or both equations have real roots.
(C) one of equations have real roots and other equation have imaginary roots
(D) at least one of the equations have imaginary roots.
ekuk p, q, r, s  R, x 2 + px + q = 0, x 2 + rx + s = 0 bl izdkj gS fd 2 (q + s) = pr rc
(A*) lehdj.k dk de ls de ,d okLrfod ewy gSA
(B) ;k rks nksuksa lehdj.kksa ds dkYifud ewy gS ;k nksuksa lehdj.kksa ds okLrfod ewy gSA
(C) lehdj.kksa eas ls ,d okLrfod ewy j[krh gS vkSj nwljh lehdj.k dkYifud ewy j[krh gSA
(D) lehdj.kksa eas ls de ls de dkYifud ewy j[krh gSA

7. The equation, x =  2x 2 + 6x  9 has:


(A*) no solution (B) one solution (C) two solutions (D) infinite solutions
lehdj.k x =  2x 2 + 6x  9 j[krh gS&
(A) dks bZ gy ugha (B) ,d gy (C) nks gy (D) vuUr gy

8. If (2 +  – 2)x 2 + ( + 2) x < 1 for all x  R, then  belongs to the interval
 2 2 
(A) (–2, 1) (B*)  2,  (C)  , 1 (D) none of these
 5 5 
;fn (2 +  – 2)x 2 + ( + 2) x < 1  x  R gks] rks '' fdl vUrjky esa fLFkr gS&
 2 2 
(A) (–2, 1) (B)  2, 5  (C)  , 1 (D) bues a ls dks bZ ugha A
 5 

9. Let conditions C1 and C2 be defined as follows : C1 : b2 – 4ac  0, C2 : a, –b, c are of same sign. The
roots of ax 2 + bx + c = 0 are real and positive, if
(A*) both C1 and C2 are satisfied (B) only C2 is satisfied
(C) only C1 is satisfied (D) none of these
;fn izfrcU/k C1 ,oa C2 bl izdkj ifjHkkf"kr gS fd C1 : b2 – 4ac  0 ,oa C2 : a, –b, c leku fpUg ds gks] rks
ax 2 + bx + c = 0 ds ew y okLrfod ,oa /kukRed gks a xs ;fn
(A) C1 ,oa C2 nks u ks a la Urq "V gks A (B) ds o y C2 la Urq "V gks A
(C) ds o y C1 la Urq "V gks A (D) bues a ls dks bZ ugha A
x2  x  c
10. If 'x' is real, then can take all real values if :
x2  x  2c

x2  x  c
;fn 'x' okLrfod gks] rks lHkh okLrfod eku xzg .k dj ldrk gS ;fn &
x2  x  2c
(A) c  [0, 6] (B) c  [ 6, 0] (C) c  (  6)  (0, ) (D*) c  ( 6, 0)

11. If both roots of the quadratic equation (2  x) (x + 1) = p are distinct & positive, then p must lie in the
interval:
;fn f}?kkr lehdj.k (2  x) (x + 1) = p ds nksuksa ewy fHkUu&fHkUu ,oa /kukRed gks] rks 'p' fuEu vUrjky esa fLFkr gksuk
pkfg,&
(A) (2, ) (B*) (2, 9/4) (C) (– , – 2) (D) (– , )

12. If two roots of the equation (a – 1) (x 2 + x + 1)2 – (a + 1) (x 4 + x 2 + 1) = 0 are real and distinct, then 'a'
lies in the interval
(A) (–2, 2) (B*) (– , –2)  (2, ) (C) (2, ) (D) (––2)
;fn lehdj.k (a – 1) (x 2 + x + 1)2 – (a + 1) (x 4 + x 2 + 1) = 0 ds nksuksa ewy okLrfod rFkk fHkUu&fHkUu gks] rks
'a' fdl vUrjky es a fLFkr gS &
(A) (–2, 2) es a (B) (– , –2)  (2, ) es a (C) (2, ) es a (D) (––2)

Corporate Office: CG Tower, A-46 & 52, IPIA, Near City Mall, Jhalawar Road, Kota (Raj.)-324005
Website : www.resonance.ac.in | E-mail : contact@resonance.ac.in
Toll Free : 1800 200 2244 | 1800 258 5555 | CIN: U80302RJ2007PTC024029
ADVQE - 27
Quadratic Equation
13. The equations x 3 + 5x 2 + px + q = 0 and x 3 + 7x 2 + px + r = 0 have two roots in common. If the third root
of each equation is represented by x 1 and x 2 respectively, then the ordered pair (x 1, x 2) is:
lehdj.kksa x 3 + 5x 2 + px + q = 0 ,oa x 3 + 7x 2 + px + r = 0 ds nks ewy mHk;fu"B gSA ;fn izR;sd lehdj.k dk rhljk
ewy Øe'k% x 1 ,oa x 2 ls iznf'kZr fd;k tkrk gks] rks Øfer ;qXe (x 1, x 2) gS&
(A*) ( 5,  7) (B) (1,  1) (C) ( 1, 1) (D) (5, 7)

14. If a, b, c are real and a2 + b2 + c2 = 1, then ab + bc + ca lies in the interval:


;fn a, b, c okLrfod gS rFkk a2 + b2 + c2 = 1 gS rc ab + bc + ca vUrjky eas fLFkr gS&
1   1   1
(A)  , 2 (B) [0, 2] (C*)   , 1 (D)   1, 
2   2   2

PART - II : SINGLE AND DOUBLE VALUE INTEGER TYPE


Hkkx - II : ,dy ,oa f}&iw.kk±d eku izdkj ¼SINGLE AND DOUBLE VALUE INTEGER TYPE½

1. Find number of integer roots of equation x (x + 1) (x + 2) (x + 3) = 120.


lehdj.k x (x + 1) (x + 2) (x + 3) = 120 ds iw.kkZad ewyksa dh la[;k Kkr dhft,A
Ans. 2

2 2
2. Find product of all real values of x satisfying (5  2 6 ) x 3
 (5  2 6 ) x 3 = 10
2 2
x ds lHkh okLrfod ekuks a dk xq . kuQy Kkr dhft, tks (5  2 6 ) x 3
 (5  2 6 ) x 3 = 10 lUrq "V djrh gS A
Ans. 8

3. The least prime integral value of '2a' such that the roots ,  of the equation 2 x 2 + 6 x + a = 0 satisfy
 
the inequality  < 2 is
 
 
'2a' dk U;wure vHkkT; iw.kkZad eku gksxk tcfd lehdj.k 2 x 2 + 6 x + a = 0 ds ewy , vlfedk  < 2 dks
 
lUrq"V djrs gS&
Ans. 11

4. If a, b are the roots of x 2 + px + 1 = 0 and c, d are the roots of x 2 + qx + 1 = 0. Then find the value of
(a  c) (b  c) (a + d) (b + d)/(q2  p2).
;fn a, b lehdj.k x 2 + px + 1 = 0 ds ewy gS rFkk c, d lehdj.k x 2 + qx + 1 = 0 ds ewy gS rc
(a  c) (b  c) (a + d) (b + d)/(q2  p2) dk eku Kkr dhft,A

Ans. 1
5. ,  are roots of the equation  (x2 – x) + x + 5 = 0. If 1 and 2 are the two values of  for which the roots

 1  2 
     
,  are connected by the relation + = 4, then the value of  2 1 
is
   14 
 
 
2
lehdj.k  (x – x) + x + 5 = 0 ds ewy ,  gSA ;fn  ds nks eku 1 ,oa 2 gSa ftlds fy, ewy ,  lEcU/k

 1  2 
    
+ = 4 }kjk lEcfU/kr gks ] rks   2 1  dk eku gS &
   14 
 
Ans. 73  
Corporate Office: CG Tower, A-46 & 52, IPIA, Near City Mall, Jhalawar Road, Kota (Raj.)-324005
Website : www.resonance.ac.in | E-mail : contact@resonance.ac.in
Toll Free : 1800 200 2244 | 1800 258 5555 | CIN: U80302RJ2007PTC024029
ADVQE - 28
Quadratic Equation
6. Let  be the roots of the equation x2 + ax + b = 0 and  be the roots of x2 – ax + b – 2 = 0. If  = 24
1 1 1 1 5
and     , then find the value of a.
    6
ekuk  lehdj.k x2 + ax + b = 0 ds ewy gS rFkk  lehdj.k x2 – ax + b – 2 = 0 ds ewy gSA ;fn  = 24 rFkk
1 1 1 1 5
    , rks a dk eku Kkr dhft,A
    6
Ans. 10

7. The least value of expression x 2 + 2 xy + 2 y2 + 4 y + 7 is:


O;atd x 2 + 2 xy + 2 y2 + 4 y + 7 dk U;wure eku gS&
Ans. 3

8. If a > b > 0 and a3 + b3 + 27ab = 729 then the quadratic equation ax 2 + bx – 9 = 0 has roots
,  ( < ). Find the value of 4 – a.
;fn a > b > 0 vkS j a 3 + b 3 + 27ab = 729 gks rFkk f}?kkr lehdj.k ax 2 + bx – 9 = 0 ds ew y
,  ( < ) gS rc 4 – a dk eku Kkr dhft,A
Ans. 13

9. Let  and  be roots of x2 – 6(t2 – 2t + 2)x – 2 = 0


a100  2a 98
with  > . If an = n – n for n  1, then find the minimum value of a 99
(where t  R)

ekuk  o lehdj.k x2 – 6(t2 – 2t + 2)x – 2 = 0 ds ewy gS rFkk  > gSA ;fn an = n – n , n  1 ds fy, gks rc
a100  2a 98
a 99
dk U;wure eku Kkr dhft,A (tgk¡ t  R)
Ans. 6

10. If , , ,  are the roots of the equation x 4  Kx 3 + Kx 2 + Lx + M = 0, where K, L & M are real numbers,
then the minimum value of 2 +  2 + 2 + 2 is – n. Find the value of n.
;fn , , ,  lehdj.k x 4  Kx 3 + Kx 2 + Lx + M = 0 ds ewy gS tgk¡ K, L vkSj M okLrfod la[;k,sa gS rc
2 +  2 + 2 + 2 dk U;uwre eku – n gS rc n dk eku Kkr dhft,A
Ans. 1

2x
11. Consider y = , where x is real , then the range of expression y 2 + y  2 is [a, b]. Find b – 4a.
1  x2

2x
ekuk fd y = 1  x 2 , tgk¡ x okLrfod gS rc O;atd y2 + y  2 dk ifjlj [a, b] gS rc b – 4a dk eku Kkr dhft,A
Ans. 9
12. If the roots of the equation x 3 + Px 2 + Qx  19 = 0 are each one more than the roots of the equaton
x 3  Ax 2 + Bx  C = 0, where A, B, C, P & Q are constants, then the value of A + B + C is equal to :
;fn lehdj.k x 3 + Px 2 + Qx  19 = 0 ds ewy lehdj.k x 3  Ax 2 + Bx  C = 0 ds izR;sd ewy ls ,d vf/kd gks]
tgk¡ A, B, C, P ,oa Q vpj gSa] rks A + B + C dk eku gS&
Ans. 18

13. If one root of the equation t2 – (12x)t – (f(x) + 64x) = 0 is twice of other, then find the maximum value of the
function f(x), where x  R.
;fn lehdj.k t2 – (12x)t – (f(x) + 64x) = 0 dk ,d ewy nwljs dk nksxquk gS rks f(x) dk vf/kdre eku Kkr dhft,] tcfd
x  R.
Ans. 32
Corporate Office: CG Tower, A-46 & 52, IPIA, Near City Mall, Jhalawar Road, Kota (Raj.)-324005
Website : www.resonance.ac.in | E-mail : contact@resonance.ac.in
Toll Free : 1800 200 2244 | 1800 258 5555 | CIN: U80302RJ2007PTC024029
ADVQE - 29
Quadratic Equation
14. Let P(x) = 4x2 + 6x + 4 and Q(y) = 4y2 – 12y + 25. If x, y satisfy equation P(x).Q(y) = 28, then find the value
of 11y – 26x.
ekuk P(x) = 4x2 + 6x + 4 vkSj Q(y) = 4y2 – 12y + 25 gS ;fn x, y lehdj.k P(x).Q(y) = 28 ds lUrq"V djrk gS rc
11y – 26x dk eku Kkr dhft,A
Ans. 36

15. The values of k, for which the equation x 2 + 2 (k  1) x + k + 5 = 0 possess atleast one positive root, are
(– , – b]. Find value of b.
lehdj.k x 2 + 2 (k  1) x + k + 5 = 0 dk de ls de ,d ewy /kukRed gksus ds fy, 'k' ds ekuksa dk leqPp;
(– , – b] gS rc b dk eku Kkr dhft,A
Ans. 1

16. If x and y both are non-negative integral values for which (xy – 7) 2 = x 2 + y2, then find the sum of all
possible values of x.
;fn x rFkk y nksuksa v_.kkRed iw.kkZad eku gS ftlds fy, (xy – 7)2 = x 2 + y2 rc x ds lHkh laHkkfor ekuksa dk ;ksxQy
Kkr dhft,A
Ans. 14

17. Find the least value of 7a for which at least one of the roots of the equation x2 – (a – 3) x + a = 0 is greater
than 2.
7a dk U;w u re eku Kkr dhft, ftlds fy, lehdj.k x2 – (a – 3) x + a = 0 ds ew yks a es a ls de ls de ,d 2 ls
vf/kd gSA
Ans. 63

18. If the quadratic equations 3x 2 + ax + 1 = 0 & 2x 2 + bx + 1 = 0 have a common root, then the value of the
expression 5ab  2a2  3b2 is
;fn f}?kkr lehdj.kksa 3x 2 + ax + 1 = 0 vkSj 2x 2 + bx + 1 = 0 dk ,d ewy mHk;fu"B gks] rks O;atd
5ab  2a2  3b2 dk eku gS
Ans. 1

19. The equations x 2  ax + b = 0, x 3  px 2 + qx = 0, where a, b, p, q  R – {0} have one common root & the
ap
second equation has two equal roots. Find value of .
qb
lehdj.ksa x 2  ax + b = 0, x 3  px 2 + qx = 0, tgk¡ a, b, p, q  R – {0} dk ,d mHk;fu"B ewy gS rFkk nwl jh
ap
lehdj.k ds nks ewy cjkcj gS rc q  b dk eku Kkr dhft,A
Ans. 2
16
20. If x – y and y – 2x are two factors of the expression x 3 – 3x 2y + xy2 + y3, then  4 is
11
16
;fn x – y rFkk y – 2x O;atd x 3 – 3x 2y + xy2 + y3 ds nks xq.ku[k.M gS rc  4 dk eku gS&
11
Ans. 1

PART - III : ONE OR MORE THAN ONE OPTIONS CORRECT TYPE


Hkkx - III : ,d ;k ,d ls vf/kd lgh fodYi çdkj
1. Possible values of 'p' for which the equation (p2 – 3p + 2)x2 – (p2 – 5p + 4)x + p – p2 = 0 does not possess
more than two roots
'p' ds laHkkfor eku gksxsa ftlds fy, lehdj.k (p2 – 3p + 2)x2 – (p2 – 5p + 4)x + p – p2 = 0 nks ls vf/kd ewy ugha j[krh
gS
(A*) 0 (B) 1 (C*) 2 (D*) 4
Corporate Office: CG Tower, A-46 & 52, IPIA, Near City Mall, Jhalawar Road, Kota (Raj.)-324005
Website : www.resonance.ac.in | E-mail : contact@resonance.ac.in
Toll Free : 1800 200 2244 | 1800 258 5555 | CIN: U80302RJ2007PTC024029
ADVQE - 30
Quadratic Equation
2. If a, b are non-zero real numbers and ,  the roots of x 2 + ax + b = 0, then
(A) 2,  2 are the roots of x 2 – (2b – a2) x + a2 = 0
1 1
(B*) , are the roots of bx 2 + ax + 1 = 0
 

 
(C*) , are the roots of bx 2 + (2b – a2) x + b = 0
 
(D*) ( – 1), ( – 1) are the roots of the equation x 2 + x (a + 2) + 1 + a + b = 0
;fn 'a' ,oa 'b' v'kwU; okLrfod la[ ;k,¡ gks ,oa x 2 + ax + b = 0 ds ewy ,  gks] rks &
(A) x 2 – (2b – a2) x + a2 = 0 ds ew y 2,  2 gks a xs A
1 1
(B) bx 2 + ax + 1 = 0 ds ew y , gksaxsA
 

 
(C) bx 2 + (2b – a2) x + b = 0 ds ew y , gksaxsA
 
(D) lehdj.k x 2 + x (a + 2) + 1 + a + b = 0 ds ewy ( – 1), ( – 1) gksaxsA

3. If ,  are the roots of ax 2 + bx + c = 0 (a  0) and  + ,  +  are the roots of,


Ax 2 + Bx + C = 0 (A  0) for some constant , then

1 B b 1 b B  b 2  4ac B 2 4AC b2  4ac B2  4AC


(A)  =    (B*)  = a  A  (C*) = (D) =
2 A a 2   a2 A2 a2 A2

;fn lehdj.k ax 2 + bx + c = 0, (a  0) ds ewy ,  gS ,oa lehdj.k Ax 2 + Bx + C = 0, (A  0) ds ewy  ds


fdlh vpj eku ds fy,  + ,  +  gS] rks
1 B b 1 b B  b 2  4ac B 2 4AC b2  4ac B2  4AC
(A)  =    (B*)  = a  A  (C*) = (D) =
2 A a 2   a2 A2 a2 A2

4. If one root of the equation 4x2 + 2x – 1 = 0 is ‘’, then

1  5 1 5
(A*)  can be equal to (B)  can be equal to
4 4
(C*) other root is 43 – 3. (D) other root is 43 + 3
;fn lehdj.k 4x2 + 2x – 1 = 0 dk ,d ewy ‘’ gS rc
1  5 1 5
(A*)  dk eku cjkcj gks ldrk gSA (B)  dk eku gks ldrk gSA
4 4
(C*) vU; ew y 43 – 3 gS A (D) vU; ew y 43 + 3gS A

5. If ,  are roots of x2 + 3x + 1 = 0, then


;fn ,  lehdj.k x2 + 3x + 1 = 0 ds ewy gS rc
(A) (7 – ) (7 – ) = 0 (B*) (2 – ) (2 – ) = 11
2 2
2 2      
(C*)  =–2 (D*)     = 18
3  1 3  1  1       1

6. If both roots of x2 – 32x + c = 0 are prime numbers then possible values of c are
;fn lehdj.k x2 – 32x + c = 0 ds nksuksa ewy vHkkT; gS rc c ds laHkkfor eku gS
(A) 60 (B*) 87 (C*) 247 (D) 231

Corporate Office: CG Tower, A-46 & 52, IPIA, Near City Mall, Jhalawar Road, Kota (Raj.)-324005
Website : www.resonance.ac.in | E-mail : contact@resonance.ac.in
Toll Free : 1800 200 2244 | 1800 258 5555 | CIN: U80302RJ2007PTC024029
ADVQE - 31
Quadratic Equation
7. Let f(x) = x2 – a(x + 1) – b = 0, a, b  R – {0}, a + b  0. If  and  are roots of equation f(x) = 0, then the value
1 1 2
of 2 + 2 – is equal to
  a   a ab
ekuk f(x) = x2 – a(x + 1) – b = 0, a, b  R – {0}, a + b  0. ;fn  rFkk  lehdj.k f(x) = 0 ds ewy gS rc
1 1 2
2 + 2 – dk eku cjkcj gS
  a   a  ab

a a2
(A*) 0 (B*) f(a) + a + b (C) f(b) + a + b (D*) f   + +a+b
2 4

8. If f(x) is a polynomial of degree three with leading coefficient 1 such that f(1) = 1, f(2) = 4, f(3) = 9, then
3
6 6
(A*) f(4) = 22 (B*) f   =  
5 5
3
(C*) f(x) = x holds for exactly two values of x. (D*) f(x) = 0 has a root in interval (0, 1).
;fn f(x), vxzx xq.kkad 1 ds lkFk rhu ?kkr dk cgqin bl izdkj gS fd f(1) = 1, f(2) = 4, f(3) = 9 rc
3
6 6
(A*) f(4) = 22 (B*) f   =  
5 5
(C*) f(x) = x3, x ds Bhd nks eku lUrq"V gksrs gSA (D*) f(x) = 0, vUrjky (0, 1) esa ,d ewy j[krk gSA

9. Let P(x) = x32 – x25 + x18 – x11 + x4 – x3 + 1. Which of the following are CORRECT ?
(A*) Number of real roots of P(x) = 0 are zero.
(B*) Number of imaginary roots of P(x) = 0 are 32.
(C*) Number of negative roots of P(x) = 0 are zero.
(D*) Number of imaginary roots of P(x) + P(–x) = 0 are 32.
ekuk P(x) = x32 – x25 + x18 – x11 + x4 – x3 + 1 fuEu esa ls dkSulsa lgh gS?
(A*) P(x) ds okLrfod ewyksa dh la[;k 0 gSA
(B*) P(x) = 0 ds dkYifud ewyksa dh la[;k 32 gSA
(C*) P(x) = 0 ds _.kkRed ewyksa dh la[;k 'kwU; gSA
(D*) P(x) + P(–x) = 0 ds dkYifud ewyksa dh la[;k 32 gSA

10. If ,  are the real and distinct roots of x 2 + px + q = 0 and 4,  4 are the roots of x 2 – rx + s = 0, then
the equation x 2 – 4qx + 2q2 – r = 0 has always
(A*) two real roots (B) two negative roots
(C) two positive roots (D*) one positive root and one negative root
;fn lehdj.k x 2 + px + q = 0 ds ewy ,  okLrfod ,oa vleku gks vkSj lehdj.k x 2 – rx + s = 0 ds ewy 4,  4
gks] rks lehdj.k x 2 – 4qx + 2q2 – r = 0 lnSo j[krh gS&
(A) nks okLrfod ew y (B) nks _.kkRed ew y
(C) nks /kukRed ew y (D) ,d /kukRed ew y vkS j ,d _.kkRed ew y

11. x 2 + x + 1 is a factor of a x 3 + b x 2 + c x + d = 0, then the real root of above equation is


(a, b, c, d  R)
;fn a x 3 + b x 2 + c x + d = 0 tgk¡ a, b, c, d  R, dk ,d xq. ku[k.M x 2 + x + 1 gks] rks nh xbZ lehdj.k dk
okLrfod ewy gS&
(A*)  d/a (B) d/a (C) (b – a)/a (D*) (a – b)/a

12. If – 5 + i, – 5 + i, 2  2 ; ,  R are roots of x3 + 15x2 + cx + 860 = 0, c  R, then


(A*) c = 222
(B) all the three roots are imaginary
(C) two roots are imaginary but not complex conjugate of each other.
(D*) – 5 + 7i 3 , – 5 – 7i 3 are imaginary roots.
Corporate Office: CG Tower, A-46 & 52, IPIA, Near City Mall, Jhalawar Road, Kota (Raj.)-324005
Website : www.resonance.ac.in | E-mail : contact@resonance.ac.in
Toll Free : 1800 200 2244 | 1800 258 5555 | CIN: U80302RJ2007PTC024029
ADVQE - 32
Quadratic Equation
;fn – 5 + i, – 5 + i, 2  2 ; ,  R lehdj.k x3 + 15x2 + cx + 860 = 0, c  R ds ewy gS rc
(A*) c = 222
(B) lHkh rhuks a ew y dkYifud gS A
(C) nks ew y dkYifud gS ijUrq ,d nw l js ds lfEeJ la ; q Xeh ugha gS A
(D*) – 5 + 7i 3 , – 5 – 7i 3 dkYifud ew y gS A

13. Let f(x) = ax2 + bx + c > 0,  x  R or f(x) < 0,  x  R. Which of the following is/are CORRECT ?
(A*) If a + b + c > 0 then f(x) > 0,  x  R (B*) If a + c < b then f(x) < 0,  x  R
(C) If a + 4c > 2b then f(x) < 0, x  R (D*) ac > 0.
ekuk f(x) = ax2 + bx + c > 0,  x  R ;k f(x) < 0,  x  R fuEu esa ls dkSulk lgh gS?
(A*) ;fn a + b + c > 0 rc f(x) > 0,  x  R (B*) ;fn a + c < b rc f(x) < 0,  x  R
(C) ;fn a + 4c > 2b rc f(x) < 0, x  R (D*) ac > 0.

14. Let x1 <  <  <  < x4, x1 < x2 < x3. If f(x) is a cubic polynomial with real coefficients such that
(f())2 + (f())2 + (f())2 = 0, f(x1) f(x2) < 0, f(x2) f(x3) < 0 and f(x1) f(x3) > 0 then which of the following are
CORRECT ?
ekuk x1 <  <  <  < x4, x1 < x2 < x3. ;fn f(x) okLrfod xq. kkad ksa dk ?kuh; cgqin bl izdkj gS fd
(f())2 + (f())2 + (f())2 = 0, f(x1) f(x2) < 0, f(x2) f(x3) < 0 rFkk f(x1) f(x3) > 0 rc fuEu es a ls dkS u lk lgh gS?
(A*)  (x1, x2),  (x2, x3) and rFkk  (x3, x4) (B)  (x1, x3), ,  (x3, x4)
(C) ,  (x1, x2) and rFkk   (x4, ) (D*)  (x1, x3),  (x2, x3) and rFkk  (x2, x4)

15. If f(x) is cubic polynomial with real coefficients,  <  <  and x1 < x2 be such that f() = f() = f() = f(x1) =
f  (x2) = 0 then possible graph of y = f(x) is (assuming y-axis vertical)
;fn okLrfod xq.kkadksa dk ?kuh; cgqin f(x),  <  <  rFkk x1 < x2 bl izdkj gS fd f() = f() = f() = f(x1) = f(x2) = 0
rc y = f(x) dk laHkkfor vkjs[k gS (ekuk fd y-v{k m/okZ/kj gS)

(A*) (B)

(C*) (D)

3 4 5
16. Let f(x) = + + , then f(x) = 0 has
x2 x3 x4
(A*) exactly one real root in (2, 3) (B*) exactly one real root in (3, 4)
(C) 3 different roots (D) at least one negative root
3 4 5
ekuk fd f(x) = + + gks] rks f(x) = 0 ds fy, &
x2 x3 x4
(A) Bhd ,d okLrfod ew y (2, 3) es a gS A (B) Bhd ,d okLrfod ew y (3, 4) es a gS A
(C) 3 fofHkUu ew y (D) de ls de ,d _.kkRed ew y

17. If the quadratic equations ax2 + bx + c = 0 (a, b, c  R, a  0) and x2 + 4x + 5 = 0 have a common root,
then a, b, c must satisfy the relations:
(A) a > b > c (B) a < b < c
(C*) a = k; b = 4k; c = 5k (k  R, k  0) (D*) b2  4ac is negative.
;fn f}?kkr lehdj.kksa ax + bx + c = 0 (a, b, c  R, a  0) ,oa x 2 + 4x + 5 = 0 dk ,d ewy mHk;fu"B gks] rks
2

a, b, c }kjk lUrq "V gks u s okyk iz frcU/k gS &


(A) a > b > c (B) a < b < c
(C) a = k; b = 4k; c = 5k (k  R, k  0) (D) b2  4ac _.kkRed gS A

Corporate Office: CG Tower, A-46 & 52, IPIA, Near City Mall, Jhalawar Road, Kota (Raj.)-324005
Website : www.resonance.ac.in | E-mail : contact@resonance.ac.in
Toll Free : 1800 200 2244 | 1800 258 5555 | CIN: U80302RJ2007PTC024029
ADVQE - 33
Quadratic Equation
18. If the quadratic equations x 2 + abx + c = 0 and x 2 + acx + b = 0 have a common root, then the equation
containing their other roots is/are :
;fn f}?kkr lehdj.kksa x 2 + abx + c = 0 ,oa x 2 + acx + b = 0 dk ,d ewy mHk;fu"B gks] rks buds vU; ewyksa ls cuk
f}?kkr lehdj.k gS&
(A) x 2 + a (b + c) x  a2bc = 0 (B*) x 2  a (b + c) x + a2bc = 0
(C) a (b + c) x 2  (b + c) x + abc = 0 (D*) a (b + c) x 2 + (b + c) x  abc = 0

19. Consider the following statements.


S1 : The equation 2x 2 + 3x + 1 = 0 has irrational roots.
S2 : If a < b < c < d, then the roots of the equation (x – a) (x – c) + 2 (x – b) (x – d) = 0 are real and
distinct.
S3 : If x 2 + 3x + 5 = 0 and ax 2 + bx + c = 0 have a common root and a, b, c  N, then the minimum
value of (a + b + c) is 10.
S4 : The value of the biquadratic expression x 4  8 x 3 + 18 x 2  8 x + 2, when x = 2 + 3 , is 1
Which of the following are CORRECT ?
(A*) S2 and S4 are true. (B*) S1 and S3 are false.
(C) S1 and S2 are true. (D) S3 and S4 are false.
fuEufyf[kr dFkuksa ij fopkj dhft, :
S1 : lehdj.k 2x 2 + 3x + 1 = 0 ds ewy vifjes; gSA
S2 : ;fn a < b < c < d gks] rks lehdj.k (x – a) (x – c) + 2 (x – b) (x – d) = 0 ds ewy okLrfod ,oa fHkUu&fHkUu
gSa A
S3 : ;fn x 2 + 3x + 5 = 0 ,oa ax 2 + bx + c = 0 dk ,d ewy mHk;fu"B ,oa a, b, c  N gks] rks (a + b + c)
dk U;wure eku 10 gSA
S4 : prqFkZ ?kkr O;atd x 4  8 x 3 + 18 x 2  8 x + 2 dk x = 2 + 3 ij eku 1 gSA
fuEu esa ls dkSulk lgh gS?
(A*) S2 rFkk S4 lR; gS A (B*) S1 rFkk S3 xyr gS A
(C) S1 rFkk S2 lR; gS A (D) S3 rFkk S4 xyr gS A

20. If the equations x 2 + a x + 12 = 0, x 2 + b x + 15 = 0 & x 2 + (a + b) x + 36 = 0 have a common positive


root, then which of the following are true ?
(A*) ab = 56 (B*) common positive root is 3
(C*) sum of uncommon roots is 21. (D) a + b = 15.
;fn lehdj.kksa x 2 + a x + 12 = 0, x 2 + b x + 15 = 0 ,oa x 2 + (a + b) x + 36 = 0 esa ,d /kukRed ewy mHk;fu"B
gks fuEu esa ls dkSuls lgh gS&
(A*) ab = 56
(B*) mHk;fu"V /kukRed ew y 3 gS
(C*) tks ew y mHkfu"V ugha gS mudk ;ks xQy 21 gS
(D) a + b = 15.

21. If x2 + x + 1 = 0,  (–2 , 2) and 4x3 + 3x + 2c = 0 have common root then c +  can be
1 1 3
(A*) (B*) – (C) 0 (D)
2 2 2
;fn x2 + x + 1 = 0,  (–2 , 2) rFkk 4x3 + 3x + 2c = 0 dk mHk;fu"B ewy gS] rks c +  gks ldrk gS
1 1 3
(A*) (B*) – (C) 0 (D)
2 2 2

Corporate Office: CG Tower, A-46 & 52, IPIA, Near City Mall, Jhalawar Road, Kota (Raj.)-324005
Website : www.resonance.ac.in | E-mail : contact@resonance.ac.in
Toll Free : 1800 200 2244 | 1800 258 5555 | CIN: U80302RJ2007PTC024029
ADVQE - 34
Quadratic Equation

PART - IV : COMPREHENSION
Hkkx - IV : vuqPNsn (COMPREHENSION)
Comprehension # 1 (Q. No. 1 & 2)

The graph of the polynomial f(x) = px 2 – qx – r is as shown

y
V

S (x3, 0) T
x
H (x1, 0) O
(x2, 0)

1. If p = –1, x 1 = –1, x 2 = –5 then the vertex V is


(A) (–3, –4) (B) (–3, 1) (C) (–4, 3) (D*) (–3, 4)

2. r(p – q – r) is
(A) positive (B*) negative (C) zero (D) > f(x3)

vuqPNsn # 1
cgqin f(x) = px 2 – qx – r dk vkjs[k fuEukuqlkj gSA

y
V

S (x3, 0) T
x
H (x1, 0) O
(x2, 0)

1. ;fn p = –1, x 1 = –1, x 2 = –5 rks 'kh"kZ V gS &


(A) (–3, –4) (B) (–3, 1) (C) (–4, 3) (D*) (–3, 4)

2. r(p – q – r) gS &
(A) /kukRed (B*) _.kkRed (C) 'kwU; (D) > f(x3)

Comprehension # 2 (Q. No. 3 & 4)


Y

In the given figure OBC is an isosceles right triangle y = x2 + bx + c


C
in which AC is a median, then answer the following

questions :
O A B X

Corporate Office: CG Tower, A-46 & 52, IPIA, Near City Mall, Jhalawar Road, Kota (Raj.)-324005
Website : www.resonance.ac.in | E-mail : contact@resonance.ac.in
Toll Free : 1800 200 2244 | 1800 258 5555 | CIN: U80302RJ2007PTC024029
ADVQE - 35
Quadratic Equation
3. Roots of y = 0 are
(A*) {2, 1} (B) {4, 2} (C) {1, 1/2} (D) {8, 4}

4. The equation whose roots are ( + ) & ( – ), where , ( > ) are roots obtained in previous question, is
(A*) x2 – 4x + 3 = 0 (B) x2 – 8x + 12 = 0 (C) 4x2 – 8x + 3 = 0 (D) x2 – 16x + 48 = 0

vuqPNsn # 2
Y
fp=kkuqlkj OBC lef}ckgq ledks.k f=kHkqt gS tcfd AC ekf/;dk gS] rks y = x2 + bx + c
fuEufyf[kr iz'uksa ds mÙkj nhft,A C

O A B X
3. y = 0 ds ewy gS &
(A*) {2, 1} (B) {4, 2} (C) {1, 1/2} (D) {8, 4}

4. og lehdj.k ftlds ewy ( + ) ,oa ( – ) gks] tgk¡ , ( > ) fiNys iz'u ls izkIr ewy gS] gksxh&
(A*) x2 – 4x + 3 = 0 (B) x2 – 8x + 12 = 0 (C) 4x2 – 8x + 3 = 0 (D) x2 – 16x + 48 = 0

Comprehension # 3 (Q. No. 5 to 7)


Consider the equation x 4 – x 2 + 9 = 0. This can be solved by substituting x 2 = t such equations are
called as pseudo quadratic equations.
5. If the equation has four real and distinct roots, then  lies in the interval
(A) (–, –6)  (6, ) (B) (0, ) (C*) (6, ) (D) (–, –6)

6. If the equation has no real root, then  lies in the interval


(A) (–, 0) (B*) (–, 6) (C) (6, ) (D) (0, )
7. If the equation has only two real roots, then set of values of  is
(A) (–, –6) (B) (–6, 6) (C) {6} (D*) 

vuqPNsn # 3
ekuk x 4 – x 2 + 9 = 0 ,d nh xbZ lehdj.k gSA bls x 2 = t izfrLFkkfir dj gy fd;k tk ldrk gSA bl izdkj dh
lehdj.k dks {kn~e f}?kkr lehdj.k (pseudo quadratic equations) dgrs gSA

5. ;fn lehdj.k ds pkj okLrfod ,oa fHkUu&fHkUu ewy gks] rks '' fuEu vUrjky esa gS &
(A) (–, –6)  (6, ) (B) (0, ) (C*) (6, ) (D) (–, –6)
6. ;fn lehdj.k dk dksbZ okLrfod ewy ugha gks] rks '' fuEu vUrjky esa gS &
(A) (–, 0) (B*) (–, 6) (C) (6, ) (D) (0, )

7. ;fn lehdj.k ds dso y nks okLrfod ewy gks] rks '' ds ekuksa dk leqPp; gS &
(A) (–, –6) (B) (–6, 6) (C) {6} (D*) 

Comprehension # 4

To solve equation of type,


ax2m + bx2m – 1 + cx2m – 2 + ......... + kx m + ......... + cx 2 + bx + a = 0, (a  0)  ()
divide by xm and rearrange terms to obtain

 1   1   1 
a  x m  m  + b  xm 1  m 1  + c  xm 2  m2  + ......... + k = 0
 x   x   x 
Substitutions like
1 1
t=x+ or t=x– helps transforming equation into a reduced degree equation.
x x
Corporate Office: CG Tower, A-46 & 52, IPIA, Near City Mall, Jhalawar Road, Kota (Raj.)-324005
Website : www.resonance.ac.in | E-mail : contact@resonance.ac.in
Toll Free : 1800 200 2244 | 1800 258 5555 | CIN: U80302RJ2007PTC024029
ADVQE - 36
Quadratic Equation
vuqPNsn # 4

lehdj.k ax2m + bx2m – 1 + cx2m – 2 + ......... + kxm + ......... + cx2 + bx + a = 0, (a  0)  () d s


izdkj dks gy djus fy,, xm ls foHkkftr djds inksa dks O;ofLFkr djrs gS
 1   1   1 
a  x m  m  + b  xm 1  m 1  + c  xm 2  m2  + ......... + k = 0
 x   x   x 

1 1
t=x+ ;k t=x– :ikUrj.k lehdj.k dh lgk;rk ls lehdj.k dh ?kkr de dh tkrh gSA
x x

8. Roots of equation x4 – 10x3 + 26x2 – 10x + 1 = 0 are


(A) 2 ± 3,3± 2 (B*) 2 ± 3 ,3±2 2
(C) 3 ± 2 ,3±2 2 (D) 8 ± 3 ,3± 2
lehdj.k x4 – 10x3 + 26x2 – 10x + 1 = 0 ds ewy gS
(A) 2 ± 3,3± 2 (B*) 2 ± 3 ,3±2 2
(C) 3 ± 2 ,3±2 2 (D) 8 ± 3 ,3± 2

9. Roots of equation x5 – 5x4 + 9x3 – 9x2 + 5x – 1 = 0 are

3  5 1 i 3 5 3 3 i
(A*) 1, , (B) 1, ,
2 2 2 2

3 5 3 i 5  3 1 i 3
(C) 1, , (D) 1, ,
2 2 2 2
lehdj.k x5 – 5x4 + 9x3 – 9x2 + 5x – 1 = 0 ds ewy gS
3  5 1 i 3 5 3 3 i
(A*) 1, , (B) 1, ,
2 2 2 2

3 5 3 i 5  3 1 i 3
(C) 1, , (D) 1, ,
2 2 2 2
10. Roots of equation x6 – 4x4 + 4x2 – 1 = 0 are

1  i 5 1  5 1  5 1  i 5
(A) ± 1, , (B) ± 1, ,
2 2 2 2

1  5 1  5 1  5 1  i 5
(C*) ± 1, , (D) ± 1, , .
2 2 2 2
lehdj.k x6 – 4x4 + 4x2 – 1 = 0 ds ewy gS
1  i 5 1  5 1  5 1  i 5
(A) ± 1, , (B) ± 1, ,
2 2 2 2

1  5 1  5 1  5 1  i 5
(C*) ± 1, , (D) ± 1, , .
2 2 2 2

Corporate Office: CG Tower, A-46 & 52, IPIA, Near City Mall, Jhalawar Road, Kota (Raj.)-324005
Website : www.resonance.ac.in | E-mail : contact@resonance.ac.in
Toll Free : 1800 200 2244 | 1800 258 5555 | CIN: U80302RJ2007PTC024029
ADVQE - 37
Quadratic Equation

 Marked Questions may have for Revision Questions.


 fpfUgr iz ' u nks g jkus ;ks X; iz ' u gS A

PART - I : JEE (ADVANCED) / IIT-JEE PROBLEMS (PREVIOUS YEARS)


Hkkx - I : JEE (ADVANCED) / IIT-JEE ¼fiNys o"kksZ½ ds iz'u
* Marked Questions may have more than one correct option.
* fpfUgr iz ' u ,d ls vf/kd lgh fodYi okys iz ' u gS -

1. Let a, b, c be the sides of a triangle (No two of them are equal) and  R. If the roots of the equation
x2 + 2(a + b + c)x + 3 (ab + bc + ca) = 0 are real, then [IIT-JEE-2006, (3, –1)/184]
ekuk ,d f=kHkqt dh Hkqtk,¡ a, b, c gS (buesa ls dksbZ Hkh nks cjkcj ugha gSa) ,oa  R A ;fn lehdj.k
x2 + 2(a + b + c)x + 3 (ab + bc + ca) ds ew y okLrfod gks ] rks

4 5 1 5 4 5
(A*)  < (B)  > (C)   ,  (D)   , 
3 3 3 3 3 3

2. If roots of the equation x2 – 10ax – 11b = 0 are c and d and those of x2 – 10cx – 11d = 0 are a and b, then find
the value of a + b + c + d. (where a, b, c, d are all distinct numbers) [IIT-JEE-2006, (6, 0)/184]
2 2
;fn lehdj.k x – 10ax – 11b = 0 ds ewy c ,oa d rFkk x – 10cx – 11d = 0 ds ewy a ,oa b gks] rks a + b + c + d
dk eku Kkr dhft, ( tgk¡ a, b, c, d lHkh fHkUu&fHkUu la[ ;k,¡ gS)A
Ans. 1210

3. Let ,  be the roots of the equation x2 – px + r = 0 and , 2 be the roots of the equation x2 – qx + r = 0.
2
Then the value of r is [IIT-JEE 2007, Paper-1, (3, –1)/ 81]

ekuk fd lehdj.k x2 – px + r = 0 ds ewy ,  gSa rFkk lehdj.k x2 – qx + r = 0 ds ewy , 2 gSa rks r dk eku gS&
2
2 2
(A) (p – q) (2q – p) (B) (q – p) (2p – q)
9 9
2 2
(C) (q – 2p) (2q – p) (D*) (2p – q) (2q – p)
9 9

1
4. Let a, b, c, p, q be real numbers. Suppose ,  are the roots of the equation x2 + 2px + q = 0 and , are

the roots of the equation ax2 + 2bx + c = 0, where 2  {–1, 0, 1} [IIT-JEE 2008, Paper-2, (3, –1)/ 81]
STATEMENT -1 : (p2 – q) (b2 – ac)  0
and
STATEMENT-2 : b  pa or c  qa
(A) STATEMENT-1 is True, STATEMENT-2 is True ; STATEMENT-2 is a correct explanation for
STATEMENT-1
(B*) STATEMENT-1 is True, STATEMENT-2 is True ; STATEMENT-2 is NOT a correct explanation for
STATEMENT-1
(C) STATEMENT-1 is True, STATEMENT-2 is False
(D) STATEMENT-1 is False, STATEMENT-2 is True

Corporate Office: CG Tower, A-46 & 52, IPIA, Near City Mall, Jhalawar Road, Kota (Raj.)-324005
Website : www.resonance.ac.in | E-mail : contact@resonance.ac.in
Toll Free : 1800 200 2244 | 1800 258 5555 | CIN: U80302RJ2007PTC024029
ADVQE - 38
Quadratic Equation
ekuk a, b, c, p, q okLrfod la[;k,¡ gSA ekuk lehdj.k x2 + 2px + q = 0 ds ewy (roots) ,  gSa vkSj lehdj.k
1
ax2 + 2bx + c = 0 ds ewy , gSa] tgk¡ 2  {–1, 0, 1}

dFku-1 : (p2 – q) (b2 – ac)  0


vkSj
dFku-2 : b  pa vFkok c  qa
(A) dFku-1 lR; gS] dFku-2 lR; gS; dFku-2, dFku-1 dk lgh Li"Vhdj.k gSA
(B*) dFku-1 lR; gS] dFku-2 lR; gS; dFku-2, dFku-1 dk lgh Li"Vhdj.k ughas gSA
(C) dFku-1 lR; gS] dFku-2 vlR; gSA
(D) dFku-1 vlR; gS] dFku-2 lR; gSA
IIT-JEE - 2009

5. The smallest value of k, for which both the roots of the equation x2 – 8kx + 16(k2 – k + 1) = 0 are real, distinct
and have values at least 4, is [IIT-JEE 2009, Paper-2, (4, –1)/ 80]
2 2
k dk og U;wure eku ftlds fy, lehdj.k x – 8kx + 16(k – k + 1) = 0 ds nksuksa ewy okLrfod ,oa fHkUu gks rFkk çR;sd
dk eku de ls de 4 gksaA
Ans. 2
IIT-JEE - 2010

6. Let p and q be real numbers such that p  0, p3  q and p3  – q. If  and  are nonzero complex numbers
 
satisfying  +  = – p and 3 + 3 = q, then a quadratic equation having and as its roots is
 
ekuk p rFkk q ,slh okLrfod la[;k,¡ gSa ftuds fy, p  0, p3  q rFkk p3  – q. ;fn  +  = – p rFkk 3 + 3 = q dks

lUrq"V djus okyh lfEeJ la[;k,¡  rFkk  gSa tks 'kwU;srj (non zero) gSa rks ,d f}?kkrh; lehdj.k ftlds ewy  rFkk


fuEu gS

(A) (p3 + q) x2 – (p3 + 2q)x + (p3 + q) = 0 (B*) (p3 + q) x2 – (p3 – 2q)x + (p3 + q) = 0
(C) (p3 – q) x2 – (5p3 – 2q)x + (p3 – q) = 0 (D) (p3 – q) x2 – (5p3 + 2q)x + (p3 – q) = 0
[IIT-JEE 2010, Paper-1, (3, –1)/ 84]
a10  2a 8
7. Let  and  be the roots of x2 – 6x – 2 = 0, with  >  . If an = n – n for n  1, then the value of
2a 9
is
(A) 1 (B) 2 (C*) 3 (D) 4
a10  2a 8
eku yks lehdj.k x2 – 6x – 2 = 0 ds ewy  rFkk  gS] tgk¡  > gSA ;fn n  1 ds fy;s an = n – n gS] rks 2a 9
dk eku gS& [IIT-JEE 2011, Paper-1, (3, –1), 80]
8. A value of b for which the equations [IIT-JEE 2011, Paper-2, (3, –1), 80]
x2 + bx – 1 = 0
x2 + x + b = 0
have one root in common is
b ds fdl eku ds fy;s lehdj.kksa
x2 + bx – 1 = 0
x2 + x + b = 0
dk ,d ewy mHk;fu"B (common) gksxk
(A) – 2 (B*) – i 3 (C) i 5 (D) 2
Corporate Office: CG Tower, A-46 & 52, IPIA, Near City Mall, Jhalawar Road, Kota (Raj.)-324005
Website : www.resonance.ac.in | E-mail : contact@resonance.ac.in
Toll Free : 1800 200 2244 | 1800 258 5555 | CIN: U80302RJ2007PTC024029
ADVQE - 39
Quadratic Equation
9. The quadratic equation p(x) = 0 with real coefficients has purely imaginary roots. Then the equation
p(p(x)) = 0 has [JEE (Advanced) 2014, Paper-2, (3, –1)/60]
(A) only purely imaginary roots (B) all real roots
(C) two real and two purely imaginary roots (D) neither real nor purely imaginary roots
okLrfod xq.kkadksa okys f}?kkr lehdj.k (quadratic equation) p(x) = 0 ds ewy iw.kZr;k dkYifud gSA rc lehdj.k
p(p(x)) = 0 ds [JEE (Advanced) 2014, Paper-2, (3, –1)/60]
(A) ds o y iw . kZ r ;k dkYifud ew y gS a A (B) lHkh ew y okLrfod gS a A
(C) nks okLrfod vkS j nks iw . kZ r ;k dkYifud ew y gS a A (D*) ew y u rks okLrfod gS a u gh iw . kZ r ;k dkYifud gS a A

10. Let S be the set of all non-zero real numbers  such that the quadratic equation x 2 – x +  = 0 has two
distinct real roots x 1 and x 2 satisfying the inequality |x 1 – x 2| < 1. Which of the following intervals is(are)
a subset(s) of S ? [JEE (Advanced) 2015, P-2 (4, –2)/ 80]
ekuk fd S mu lHkh 'kwU;srj (non-zero) okLrfod la[;kvksa  dk leqPPk; (set) gS ftuds fy, f}?kkrh lehdj.k
x 2 – x +  = 0 ds nks fofHkUu okLrfod ewy x 1 vkSj x 2 vlfedk |x 1 – x 2| < 1 dks larq"V djrs gSA fuEufyf[kr varjkyksa
esa ls dkSu lk (ls) leqPp; S ds mileqPp; gS (gSa) ? [JEE (Advanced) 2015, P-2 (4, –2)/ 80]
 1 1   1   1   1 1
(A*)  – , – 
 (B)  – , 0  (C)  0, 
 (D*)  , 
 2 5  5   5  5 2

PART - II : JEE (MAIN) / AIEEE PROBLEMS (PREVIOUS YEARS)


Hkkx - II : JEE (MAIN) / AIEEE ¼fiNys o"kksZ½ ds iz'u

 P Q
1. In a triangle PQR, R = . If tan   and tan   are the roots of ax2 + bx + c = 0, a  0, then :
2 2 2

 P Q
,d f=kHkqt PQR esa R = gSA ;fn ax2 + bx + c = 0, a  0 ds ewy tan  2  ,oa tan  2  gks] rks&
2    
[AIEEE-2005 (3, 0), 225]
(1) b = a + c. (2) b = c. (3*) c = a + b. (4) a = b + c.

2. The value of 'a' for which the sum of the squares of the roots of the equation x2 – (a –2) x– a –1 = 0 assume
the least value is - [AIEEE-2005 (3, 0), 225]
2
'a' dk eku ftlds fy, lehdj.k x – (a –2) x– a –1 = 0 ds ew yks a ds oxks Z dk ;ks x U;w u re gks r k gS &
[AIEEE-2005 (3, 0), 225]
(1*) 1 (2) 0 (3) 3 (4) 2

3. If both the roots of the quadratic equation x2 –2kx + k2 +k –5 = 0 are less than 5, then 'k' lies in the interval
[AIEEE-2005 (4½, 0), 225]
(1) (5, 6) (2) (6,  ) (3*) (–  ,4) (4) [4, 5]
;fn f}?kkr lehdj.k x2 –2kx + k2 +k –5 = 0 ds nksuks ewy 5 ls NksVs gS rks 'k' fdl vUrjky esa fLFkr gksxk&
[AIEEE-2005 (4½, 0), 225]
(1) (5, 6) es a (2) (6,  ) es a (3*) (–  ,4) es a (4) [4, 5] es a

4. If the roots of the quadratic equation x2 + px + q = 0 are tan 30° and tan 15° respectively, then the value
of 2 + q – p is : [AIEEE-2006(3, –1), 120]
;fn f}?kkr lehdj.k x2 + px + q = 0 ds ewy tan 30° ,oa tan 15° gks] rks 2 + q – p dk eku gS&
(1*) 3 (2) 0 (3) 1 (4) 2

Corporate Office: CG Tower, A-46 & 52, IPIA, Near City Mall, Jhalawar Road, Kota (Raj.)-324005
Website : www.resonance.ac.in | E-mail : contact@resonance.ac.in
Toll Free : 1800 200 2244 | 1800 258 5555 | CIN: U80302RJ2007PTC024029
ADVQE - 40
Quadratic Equation
5. All the values of 'm' for which both roots of the equation x2 – 2mx + m2 – 1 = 0 are greater than – 2 but less
than 4 lie in the interval : [AIEEE-2006 (3, –1), 120]
lehdj.k x2 – 2mx + m2 – 1 = 0 ds nksuksa ewy – 2 ls cM+s ijUrq 4 ls NksVs gksus ds fy, 'm' ds leLr ekuksa dk vUrjky
gS& [AIEEE-2006 (3, –1), 120]

(1) m > 3 (2*) – 1< m < 3 (3) 1 < m < 4 (4) – 2 < m < 0

3x 2  9x  17
6. If 'x' is real, the maximum value of is - [AIEEE-2006 (3, –1), 120]
3x 2  9x  7

3x 2  9 x  17
;fn 'x' okLrfod gS rks dk vf/kdre eku gS - [AIEEE-2006 (3, –1), 120]
3 x 2  9x  7

17 1
(1*) 41 (2) 1 (3) (4)
7 4

7. If the difference between the roots of the equation x2 + ax + 1 = 0 is less than 5 , then the set of possible
values of 'a' is [AIEEE-2007, (3, –1), 120]
;fn lehdj.k x2 + ax + 1 = 0 ds ewyksa dk vUrj 5 ls NksVk gks] rks 'a' ds laHkkfor ekuksa dk leqPp; gS&
[AIEEE-2007, (3, –1), 120]
(1*) (–3, 3) (2) (–3, ) (3) (3, ) (4) (–, -3)

8. The quadratic equations x2 – 6x +a = 0 and x2– cx + 6 = 0 have one root in common. The other roots of the
first and second equations are integers in the ratio 4 : 3. Then the common root is
[AIEEE-2008, (3, –1), 105]
2 2
f}?kkr lehdj.k x – 6x +a = 0 rFkk x – cx + 6 = 0 dk ,d ewy mHk;fu"B gSA izFke rFkk f}rh; lehdj.k ds vU;
ewy iw. kk±d gS] tks 4 : 3 ds vuqikr esa gS] rks mHk;fu"B ewy gksxk& [AIEEE-2008, (3, –1), 105]
(1) 4 (2) 3 (3*) 2 (4) 1

9. How many real solution does the equation x7 + 14x5 + 16x3 + 30x – 560 = 0 has? [AIEEE-2008, (3, –1), 105]
lehdj.k x7 + 14x5 + 16x3 + 30x – 560 = 0 ds fdrus okLrfod gy gS? [AIEEE-2008, (3, –1), 105]
(1*) 1 (2) 3 (3) 5 (4) 7

10. Sachin and Rahul attempted to solve a quadratic equaiton. Sachin made a mistake in writing down the
constant term and ended up in roots (4, 3). Rahul made a mistake in writing down coefficient of x to get roots
(3, 2). The correct roots of equation are : [AIEEE- 2011, II, (4, –1), 120]
lfpu rFkk jkgqy ,d f}?kkr lehdj.k dks gy djus dk iz;kl djrs gaSA lfpu us vpj in fy[kus esa xyrh dh rFkk ewy
(4, 3) ik,A jkgqy us x dk xq.kkad xyr fy[kk rFkk ewy (3, 2) ik,A lehdj.k ds lgh ewy gS %
(1*) 6, 1 (2) 4, 3 (3) –6 , –1 (4) –4 , –3

11. Let for a  a1  0, f(x) = ax2 + bx + c, g(x) = a1x2 + b1x + c1 and p(x) = f(x) – g(x). If p(x) = 0 only f or
x = –1 and p(–2) = 2, then the value of p(2) is : [AIEEE- 2011, II, (4, –1), 120]
ekuk a  a1  0, f(x) = ax2 + bx + c, g(x) = a1x2 + b1x + c1 rFkk p(x) = f(x) – g(x) . ;fn dsoy x = – 1 ds fy,
p(x) = 0 rFkk p(–2) = 2 gS] rks p(2) dk eku gS% [AIEEE- 2011, II, (4, –1), 120]
(1) 3 (2) 9 (3) 6 (4*) 18

12. The equation esinx – e–sinx – 4 = 0 has : [AIEEE- 2012 (4, –1), 120]
(1) infinite number of real roots (2*) no real roots
(3) exactly one real root (4) exactly four real roots

Corporate Office: CG Tower, A-46 & 52, IPIA, Near City Mall, Jhalawar Road, Kota (Raj.)-324005
Website : www.resonance.ac.in | E-mail : contact@resonance.ac.in
Toll Free : 1800 200 2244 | 1800 258 5555 | CIN: U80302RJ2007PTC024029
ADVQE - 41
Quadratic Equation
lehdj.k esinx – e–sinx – 4 = 0 ds %
(1) vuUr okLrfod ewy gSaA (2*) dksbZ okLrfod ewy ugha gSaA
(3) ek=k ,d okLrfod ewy gSA (4) ek=k pkj okLrfod ewy gSaA

13 . If the equations x2 + 2x + 3 = 0 and ax2 + bx + c = 0, a,b,c R, have a common root, then a : b : c is
[AIEEE - 2013, (4, –1), 120]
;fn lehdj.kksa x2 + 2x + 3 = 0 rFkk ax2 + bx + c = 0, a,b,c R, dk ,d ewy mHk;fu"B gS] rks a : b : c gS&
(1*) 1 : 2 : 3 (2) 3 : 2 : 1 (3) 1 : 3 : 2 (4) 3 : 1 : 2
[AIEEE - 2013, (4, –1),360]

14. If a  R and the equation – 3(x – [x])2 + 2 (x – [x]) + a2 = 0 (where [x] denotes the greatest integer  x) has
no intgeral solution, then all possible values of a lie in the interval :
[JEE(Main) 2014, (4, – 1), 120]
2 2
;fn a  R rFkk lehdj.k –3(x – [x]) + 2 (x – [x]) + a = 0 (tgk¡ [x] ml cM+s ls iw.kkZ±d dks n'kkZrk gS tks  x gS) dk
dksbZ iw.kkZ±dh; gy ugh gS] rks a ds lHkh laHko eku ftl varjky esa fLFkr gSa] og gSµ
[JEE(Main) 2014, (4, – 1), 120]
(1) (–2, –1) (2) (–, –2)  (2, ) (3*) (–1, 0)  (0, 1) (4) (1, 2)
1 1
15. Let  and  be the roots of equation px2 + qx + r = 0, p  0. If p, q ,r are in the A.P. and  = 4, then the
 
value of | – | is : [JEE(Main) 2014, (4, – 1), 120]
1 1
ekuk  rFkk  lehdj.k px2 + qx + r = 0, p  0 ds ewy gSA ;fn p, q ,r lekUrj Js<+h esa gSa rFkk    = 4 gS] rks
| – | dk eku gSµ [JEE(Main) 2014, (4, – 1), 120]

34 2 13 61 2 17
(1) (2*) (3) (4)
9 9 9 9
a10 – 2a 8
16. Let  and  be the roots of equation x2 – 6x – 2 = 0. If an = n – n, for n  1, then the value of is
2a 9
equal to :
(1) 6 (2) – 6 (3*) 3 (4) –3
a10 – 2a 8
ekuk  rFkk  f}?kkr lehdj.k x2 – 6x – 2 = 0 ds ewy gSaA ;fn n  1 ds fy, an = n – n gS] rks dk eku
2a 9
gS : [JEE(Main) 2015, (4, – 1), 120]
(1) 6 (2) – 6 (3*) 3 (4) –3

SUBJECTIVE QUESTIONS
fo"k;kRed iz 'u ¼SUBJECTIVE QUESTIONS½

1. Prove that roots of a2x 2 + (b2 + a2  c2) x + b2 = 0 are not real, if a + b > c and a  b < c.
(where a, b, c are positive real numbers)
;fn a + b > c ,oa a  b < c gks] rks fl) dhft, fd a2x 2 + (b2 + a2  c2) x + b2 = 0 ds ewy okLrfod ugha
gSA ¼tgk¡ a, b, c /kukRed okLrfod la[ ;k,a gSa½
a b
2. Find the condition that + = m may have roots equal in magnitude but opposite in sign.
( x  a) ( x  b)

Corporate Office: CG Tower, A-46 & 52, IPIA, Near City Mall, Jhalawar Road, Kota (Raj.)-324005
Website : www.resonance.ac.in | E-mail : contact@resonance.ac.in
Toll Free : 1800 200 2244 | 1800 258 5555 | CIN: U80302RJ2007PTC024029
ADVQE - 42
Quadratic Equation
a b
+ = m ds ew y ekika d es a cjkcj ijUrq foijhr fpUg ds gks u s ds fy, iz frcU/k Kkr dhft,A
( x  a) ( x  b)
Ans. a + b = 0, m  (– , – 2) (0, )
or m = – 1, ab > 0
1 4 4 1 1
3. Solve the inequality, – + – < .
x 1 x  2 x  3 x4 30
1 4 4 1 1
vlfedk – + – < dks gy dhft,A
x 1 x  2 x  3 x  4 30
Ans. (– , – 2)  (– 1, 1)  (2, 3)  (4, 6)  (7, )

4. If three real and distinct numbers a, b, c are in G.P. (i.e., b 2 = ac) and a + b + c = x b, then prove that
x <  1 or x > 3.
;fn rhu vleku okLrfod la[ ;k,¡ a, b, c bl izdkj gS fd ;s ,d xq. kksÙkj Js<+h esa gS (vFkkZr~ b2 = ac)] rFkk ;fn
a + b + c = x b gS ] rks fl) dhft, fd x <  1 ;k x > 3.

5. If Vn = n +  n , where  are roots of equation x 2 + x – 1 = 0. Then prove that V n + Vn–3 = 2 Vn–2 and
hence evaluate V7 (n is a whole number)
;fn Vn = n +  n , tgk¡  lehdj.k x 2 + x – 1 = 0 ds ewy gS rc fl) dhft, Vn + Vn–3 = 2 Vn–2 vr% V7 Kkr
dhft,& (n iw.kZ la[;k gSA)
Ans. –29

6. Find all ‘m’ for which f (x)  x 2  (m  3) x + m > 0 for all values of ‘x’ in [1, 2].
vUrjky [1, 2] esa 'x' ds lHkh ekuksa ds fy, f (x)  x 2  (m  3) x + m > 0 gksus ds fy, 'm' ds lHkh eku Kkr dhft,A
Ans. (, 10)

7. Find the values of a, for which the quadratic expression ax 2 + (a – 2) x – 2 is negative for exactly two
integral values of x.
'a' ds os eku Kkr dhft;s ftuds fy;s f}?kkr O;atd ax 2 + (a – 2) x – 2, 'x' ds Bhd nks iw.kk±d ekuksa ds fy;s _.kkRed
gksA
Ans. [1, 2)

8. (i) Find the roots of 4x 3 + 20x 2 – 23x + 6 = 0 if two of its roots are equal.
lehdj.k 4x 3 + 20x 2 – 23x + 6 = 0 ds ewy Kkr dhft,] ;fn blds nks ewy leku gSaA
1 1
Ans. , ,–6
2 2
(ii) Solve for real values of 'x' : 'x' ds okLrfod ekuks a ds fy, gy dhft, &
x 2 – 2a |x – a| – 3a2 = 0, a  0
Ans. x = a (1 – 2 ), x = a ( 6 – 1)
3
 1  1
(iii) Find the number of real roots of  x   +  x   = 0
 x   x

3
 1  1
 x   +  x   = 0 ds okLrfod ewyksa dh la[;k Kkr dhft,A
 x  x
Ans. 0

9. If ,  are roots of the equation x 2 – 34x + 1 = 0, evaluate 4 4 4


 –  , where . denotes the principal
value.

Corporate Office: CG Tower, A-46 & 52, IPIA, Near City Mall, Jhalawar Road, Kota (Raj.)-324005
Website : www.resonance.ac.in | E-mail : contact@resonance.ac.in
Toll Free : 1800 200 2244 | 1800 258 5555 | CIN: U80302RJ2007PTC024029
ADVQE - 43
Quadratic Equation
;fn lehdj.k x 2 – 34x + 1 = 0 ds ewy ,  gks] rks 4  – 4  Kkr dhft,] tgk¡ 4 . eq[ ; eku dks iznf'kZr djrk
gS A
Ans. : ± 2

10. Find the values of 'a' for which the equation

x 2
 2
    2
 x  2  a  3 x 2  x  2 x 2  x  1  a  4  x 2  x  1  0 has at least one real root.
'a' ds lHkh ekuksa dks Kkr dhft, ftuds fy, lehdj.k

x 2

2
    
2
 x  2  a  3 x 2  x  2 x 2  x  1  a  4  x 2  x  1  0 dk de ls de ,d okLrfod ewy fo|eku gksA

19
Ans. 5<a<
3
11. Show that the quadratic equation x 2 + 7x – 14(q2 + 1) = 0 where q is an integer, has no integral roots.
n'kkb;s fd f}?kkr lehdj.k x 2 + 7x – 14(q2 + 1) = 0 tgk¡ q ,d iw. kk±d gS] ds iw. kk±d ewy ugha gSA

12.   
Find the integral values of 'a' for which the equation x 4  a 2  5a  6 x 2  a 2  3a  2 = 0 has only real 
roots.

  
'a' ds iw.kk±d ekuksa dks Kkr dhft, ftuds fy, lehdj.k x 4  a 2  5a  6 x 2  a 2  3a  2 = 0 dsoy okLrfod ewy 
j[krk gSA
Ans. a  {1, 2}

13. If , ,  and ,  are the roots of a i x 2  b i x  c i  0 ; i = 1, 2, 3 then show that


1
 3  a  b  c  2

              = ±   i ai i   1


 i  1  i 
;fn , ,  rFkk ,  lehdj.k ai x 2  bi x  c i  0 ; i = 1, 2, 3 ds ewy gS] rks iznf'kZr dhft, fd
1
 3  a  b  c  2

              = ±   i ai i  1


 i  1  i 

14. Suppose that a1 > a2 > a3 > a4 > a5 > a6 and


p = a1 + a2 + a3 + a4 + a5 + a6
q = a1a3 + a3a5 + a5a1 + a2a4 + a4a6 + a6a2
r = a1a3a5 + a2a4a6 ,
then show that roots of the equation 2x3 – px2 + qx – r = 0 are real.
ekuk fd a1 > a2 > a3 > a4 > a5 > a6 vkSj
p = a1 + a2 + a3 + a4 + a5 + a6
q = a1a3 + a3a5 + a5a1 + a2a4 + a4a6 + a6a2
r = a1a3a5 + a2a4a6 ,
rks iznf'kZr dhft, fd lehdj.k 2x3 – px2 + qx – r = 0 ds ewy okLrfod gSaA

Corporate Office: CG Tower, A-46 & 52, IPIA, Near City Mall, Jhalawar Road, Kota (Raj.)-324005
Website : www.resonance.ac.in | E-mail : contact@resonance.ac.in
Toll Free : 1800 200 2244 | 1800 258 5555 | CIN: U80302RJ2007PTC024029
ADVQE - 44
Quadratic Equation
15. If   + cos2,  + sin2 are the roots of x2 + 2bx + c = 0 and   cos4,   sin4 are the roots of
X2 + 2BX + C = 0, then prove that b2 – B2 = c – C.
;fn x2 + 2bx + c = 0 ds ewy + cos2, + sin2 gS] rFkk X2 + 2BX + C = 0 ds ewy cos4, sin4 gS] rks fl)
dhft, fd b2 – B2 = c – C.

16. Find the set of values of 'a' if (x 2 + x)2 + a (x 2 + x) + 4 = 0 has


(i) all four real & distinct roots.
(ii) four roots in which only two roots are real and distinct.
(iii) all four imaginary roots.
(iv) four real roots in which only two are equal.
'a' ds ekuks a dk leq Pp; Kkr dhft, ;fn (x 2 + x)2 + a (x 2 + x) + 4 = 0 ds
(i) pkjksa ewy okLrfod ,oa fHkUu&fHkUu gSaA
(ii) pkj ewy gksaxs ftuesa ls dso y nks ewy okLrfod ,oa fHkUu&fHkUu gSaA
(iii) lHkh pkjksa ewy dkYifud gaSA
(iv) pkj okLrfod ewy gSa ftuesa ls dso y nks leku gSA
 65   65 
Ans. (i) a  (– , – 4) (ii) a   ,  (iii) a    4,  (iv) a  
4  4 

17. f(x) = x 2 + bx + c , where b,c  R, if f(x) is a factor of both x 4 + 6x 2 + 25 and 3x 4 + 4x 2 + 28x + 5 then
find f(x).
f(x) = x 2 + bx + c , tgk¡ b,c  R, ;fn f(x) nksuksa x 4 + 6x 2 + 25 vkSj 3x 4 + 4x 2 + 28x + 5 dk xq.ku[k.M gks] rks
f(x) Kkr dhft,A
Ans. x 2 – 2x + 5

18. How many quadratic equations are there which are unchanged by squaring their roots ?
fdrus f}?kkr lehdj.k blizdkj gS fd muds ewyksa dk oxZ djus ij ifjofrZr ugha gksrh gS&
Ans. 4

19. Let P(x) = x5 + x2 + 1 have zeros 1, 2, 3, 4, 5 and Q(x) = x2 – 2, then find
ekuk P(x) = x5 + x2 + 1 ds 'kwU; 1, 2, 3, 4, 5 gS vkSj Q(x) = x2 – 2, rc Kkr dhft,
5 5 5
(i)  Q(i ) (ii)  Q( i ) (iii)   Q( ) Q( )
1  i  j 5
i j (iv) 2
 Q ( ) i
i 1 i1 i 1

Ans. (i) – 23 (ii) – 10 (iii) 40 (iv) 20

20. If a, b, c are non–zero, unequal rational numbers then prove that the roots of the equation
(abc2)x 2 + 3a2 cx + b2 cx – 6a2 – ab + 2b2 = 0 are rational.
;fn a, b, c v'kwU;] vleku ifjes; la[ ;k,¡ gks] rks fl) dhft, fd lehdj.k
(abc2)x 2 + 3a2 cx + b2 cx – 6a2 – ab + 2b2 = 0 ds ew y ifjes ; gS A

21. If a, b, c represents sides of a  then prove that equation x 2 – (a2 + b2 + c2)x + a2b2 + b2c2 + c2a2 = 0 has
imaginary roots.
;fn a, b, c f=kHkqt dh Hkqtk,a gks] rks fl) dhft, fd lehdj.k x 2 – (a2 + b2 + c2)x + a2b2 + b2c2 + c2a2 = 0 ds dkYifud
ewy gSA

22. Let  + i ; ,  R, be a root of the equation x 3 + qx + r = 0; q, r  R. Find a real cubic equation,
independent of  and , whose one root is 2.
;fn lehdj.k x 3 + qx + r = 0 tgk¡ q, r  R, dk ,d ewy  + i ; ,  R gSA  ,oa ls Lora=k okLrfod f=k?kkfr;
lehdj.k Kkr dhft, ftldk ,d ewy 2gksA
Ans. x 3 + qx – r = 0

Corporate Office: CG Tower, A-46 & 52, IPIA, Near City Mall, Jhalawar Road, Kota (Raj.)-324005
Website : www.resonance.ac.in | E-mail : contact@resonance.ac.in
Toll Free : 1800 200 2244 | 1800 258 5555 | CIN: U80302RJ2007PTC024029
ADVQE - 45
Quadratic Equation
23. If x 1 is a root of ax 2 + bx + c = 0, x 2 is a root of  ax 2 + bx + c = 0 where 0 < x 1 < x 2, show that the
equation ax 2 + 2 bx + 2c = 0 has a root x 3 satisfying 0 < x 1 < x 3 < x 2.
;fn ax 2 + bx + c = 0 dk ,d ewy 'x 1' rFkk  ax 2 + bx + c = 0 dk ,d ewy 'x 2' tgk¡ 0 < x 1 < x 2 gks] rks iznf'kZr
dhft, fd ax 2 + 2 bx + 2c = 0 dk ,d ewy 'x 3' bl izdkj gksxk fd 0 < x 1 < x 3 < x 2A

24. Find the number of positive real roots of x 4 – 4x – 1 = 0.


lehdj.k x 4 – 4x – 1 = 0 ds /kukRed okLrfod ewyksa dh la[;k Kkr dhft,A
Ans. 1

25. If (1 + k) tan2x – 4 tan x – 1 + k = 0 has real roots tan x 1 and tan x 2, where tan x 1  tan x 2 , then find k.
;fn (1 + k) tan2x – 4 tan x – 1 + k = 0 ds okLrfod ewy tan x 1 vkSj tan x 2 gS, tgk¡ tan x 1  tan x 2 , rc k dk
eku Kkr dhft,A
Ans.   
5,  1  1, 5 
26. Let  2 be the discriminant and ,  be the roots of the equation ax 2 + bx + c = 0. Then
find equation whose roots are 2a +  and 2a  – 
ekuk 2 ,d foospd gS rFkk lehdj.k ax 2 + bx + c = 0 ds ewy ,  gS rc lehdj.k Kkr dhft, ftlds ewy
2a +  vkS j 2a  – gS &
Ans. x 2 + 2b x + b2 = 0 or ;k x 2 + 2bx – 3b2 + 16 ac = 0

e e   e e
27. Prove that    0 has one real root in (e, ) and other in (, + e).
xe x xe

e e   e e
fl) dhft, fd   0 dk ,d okLrfod ewy (e, ) esa vkSj vU; (, + e) esa gSA
xe x xe

28. If , 2 are integers, 2 is non-zero multiple of 3 and  + i, – 2 are roots of x3 + ax2 + bx – 316 = 0,
a, b,   R, then find a, b. Ans. a = 0, b = 63
;fn ,  iw . kkZ a d gS ,  , 3 dk v'kw U ; xq . k t gS vkS j l ehdj. k x 3 + ax 2 + bx – 316 = 0,
2 2

a, b,  R ds ewy  + i, – 2 gS rc a, b Kkr dhft,A


Ans. a = 0, b = 63

Corporate Office: CG Tower, A-46 & 52, IPIA, Near City Mall, Jhalawar Road, Kota (Raj.)-324005
Website : www.resonance.ac.in | E-mail : contact@resonance.ac.in
Toll Free : 1800 200 2244 | 1800 258 5555 | CIN: U80302RJ2007PTC024029
ADVQE - 46

You might also like